M

Chiudi

Derivate: teoria

Derivate

Home » Derivate: teoria

Derivate: teoria

La nozione di derivata è una delle più importanti dell’Analisi Matematica. Essa formalizza l’idea intuitiva di “tasso di variazione puntuale” di una funzione. Infatti, la derivata f'(x_0) esprime il rapporto tra la variazione di f(x) e quella di x per dei valori x “molto vicini a x_0”.

Questo strumento possiede applicazioni nella descrizione di quasi tutti i fenomeni in cui sia necessario quantificare l’evoluzione di una grandezza in funzione di un’altra, risultando dunque onnipresente nelle scienze naturali e applicate.

Questa dispensa è un’approfondita risorsa per chiunque sia interessato a comprendere i fondamenti sulle derivate, che si focalizza sui seguenti punti fondamentali:

  • Cosa sono il rapporto incrementale e la derivata e qual è il loro significato geometrico?
  • Quale relazione esiste tra continuità e derivabilità?
  • Quali sono le principali regole di derivazione e come si calcolano le derivate delle funzioni elementari?
  • Punti di non derivabilità e loro classificazione;
  • Quali relazioni esistono tra segno della derivata e monotonia di una funzione?
  • Derivate di ordine superiore e convessità.

Il testo è ulteriormente arricchito da esempi pratici, grafici ed esercizi svolti. Si rivela dunque un eccellente punto di partenza per studenti e appassionati che vogliono esplorare questo argomento essenziale.

Consigliamo la lettura delle seguenti risorse di esercizi sulle derivate:

Seganliamo inoltre il seguente materiale su argomenti di teoria collegata, di cui è possibile reperire un’esaustiva lista alla fine dell’articolo:

 

 

Autori e revisori


 
 

Prerequisiti

Leggi...

In queste note si presuppone la conoscenza di alcuni concetti di base dell’analisi matematica, in particolare del concetto di funzione, di limite e di continuità di una funzione.

 
 

Notazioni

Leggi...

\mathbb{N}=\{ 0,1, \dots \}      Insieme dei numeri naturali;
\mathbb{Z}      Insieme dei numeri interi relativi;
\mathbb{R}      Insieme dei numeri reali;
r_{f,x_0}(x)      Rapporto incrementale della funzione f in un punto x_0;
\Delta x      Incremento della variabile indipendente;
\Delta f (x_0)      Incremento della funzione f in un punto x_0 relativo all’incremento \Delta x;
f^\prime ( risp. \dfrac{df}{dx},\quad \dfrac{d}{dx}f,\quad Df)      Derivata prima della funzione f;
-A=\{ -x\colon x \in A \}      Insieme simmetrico di A \subset \mathbb{R} rispetto lo 0;
|f|      Modulo della funzione f.


 
 

Introduzione

Leggi...

Il concetto di derivata è uno dei cardini dell’analisi matematica. Esso fu sviluppato all’inizio del XVIII secolo da Sir Isaac Newton (1642 – 1727) e da Gottfried Wilhelm von Leibniz (1646 – 1716). Il significato pratico di derivata è il tasso di variazione di una certa grandezza presa in considerazione. Un esempio concreto è la variazione della posizione di un oggetto rispetto al tempo: la velocità istantanea. Il concetto di derivata ha applicazioni in svariate discipline come in fisica, in ingegneria e in generale dovunque si cerchi la variazione istantanea di una certa grandezza presa in considerazione.

Nel primo capitolo di questa dispensa introduciamo il concetto di derivata prima di una funzione. Nel secondo capitolo esponiamo le principali tecniche per calcolare la derivata di una funzione. Il terzo capitolo è dedicato allo studio delle proprietà della derivata prima. Il quarto capitolo presenta il concetto di derivata di ordine superiore di una funzione e intende descrivere le principali proprietà della derivata seconda di una funzione. Il quinto e ultimo capitolo è una raccolta di esercizi svolti in cui si guida il lettore in maniera graduale verso l’applicazione delle regole di calcolo esposte nel secondo capitolo.


 
 

Il calcolo differenziale

Definizione di rapporto incrementale.

Supponiamo di avere una funzione che rappresenta la posizione di un punto materiale su una retta al variare del tempo. Sull’asse delle ascisse abbiamo il tempo, sulle asse delle ordinate troviamo la posizione del punto materiale sulla retta. Ci si può chiedere la variazione di posizione in un certo arco di tempo, quella che comunemente viene chiamata velocità media. Supponiamo quindi che un punto materiale si trovi al tempo t_0 nella posizione s(t_0) e che al tempo t_0 + \Delta t si trovi nella posizione s(t_0 + \Delta t). Lo spazio percorso nell’intervallo temporale (t_0,t_0+\Delta t) di ampiezza \Delta t è

    \[\Delta s \coloneqq s(t_0+\Delta t)-s(t_0).\]

Definiamo dunque la velocità media (in una qualche unità di misura non specificata) del punto materiale nell’intervallo di tempo (t_0,t_0+\Delta t) come il rapporto tra lo spazio percorso e il tempo impiegato per percorrere un tale spazio, ovvero

    \[\bar{v}=\frac{\Delta s}{\Delta t} =\frac{s(t_0 + \Delta t) - s(t_0)}{\Delta t}.\]

Data una qualunque funzione y=f(x), è facile generalizzare quanto appena visto nella seguente definizione.

    \[\quad\]

Definizione 1.1 (rapporto incrementale). Siano f\colon A \subseteq \mathbb{R}\to \mathbb{R}, x_0\in A. Si definisce rapporto incrementale di f in x_{0} la funzione r_{f,x_0}\colon A \setminus \{ x_0 \} \to \mathbb{R} data da

    \[r_{f,x_0}(x)=\frac{f(x)-f(x_{0})}{x-x_0}.\]

    \[\quad\]

Il rapporto incrementale rappresenta il coefficiente angolare della retta secante al grafico della funzione f passante per i punti (x_0,f(x_0)) e (x,f(x)).

Osservazione 1.2. Nella pratica spesso si utilizza un cambio di variabile, ovvero si riscrive il rapporto incrementale in funzione dell’incremento \Delta x= x-x_0, sostituendo cioè x=x_0+\Delta x nell’espressione data sopra. Siano f\colon A \subseteq \mathbb{R}\to \mathbb{R}, x_0\in A. Supponendo che esista \delta >0 tale per cui se |\Delta x| < \delta si ha x_0+\Delta x \in A, possiamo definire la funzione \tilde{r}_{f,x_0}\colon (-\delta,\delta) \to \mathbb{R} data da

    \[\tilde{r}_{f,x_0}(\Delta x)=r_{f,x_0}(x_0+\Delta x)=\frac{f(x_0+\Delta x)-f(x_0)}{\Delta x},\]

detta anch’essa rapporto incrementale di f in x_{0}. Spesso per brevità si è soliti denotare con h l’incremento. Segnaliamo inoltre, in analogia con l’esempio della velocità media, la notazione

    \[\frac{\Delta f(x)}{\Delta x}=\frac{f(x+\Delta x)-f(x)}{\Delta x}\]

per indicare il rapporto incrementale in x.

    \[\quad\]

    \[\quad\]

    \[\quad\]

Rendered by QuickLaTeX.com

    \[\quad\]

    \[\quad\]

Figura 1: Rappresentazione grafica del rapporto incrementale come coefficiente angolare della retta secante passante per (x_0,f(x_0)) e (x_0+\Delta x,f(x_0+\Delta x)). Per \Delta x abbastanza piccolo tale retta approssima la retta tangente al grafico di f nel punto (x_0,f(x_0)) (tratteggiata in blu).

Il dominio di definizione del rapporto incrementale non è particolarmente rilevante. Quello che ci interessa veramente è il comportamento di questa funzione in un intorno (destro, sinistro o circolare) sufficientemente piccolo di x_0.


Definizione di derivata.

Diamo ora la definizioni di derivata, cf. [2, definizione 7.3]. Rimandiamo inoltre il lettore alla lettura di [1], [3] o del classico [4].

Definizione 1.3 (derivabilità). Siano f\colon A\subseteq\mathbb{R} \rightarrow \mathbb{R} e x_{0} \in A un punto di accumulazione per A. Si dice che f è derivabile in x_{0} se esiste finito il limite del rapporto incrementale:

(1)   \begin{equation*} 				f'(x_{0})=\lim_{x \to x_0} r_{f,x_0}(x)=\lim_{x \to x_0}  \dfrac{f(x)-f(x_{0})}{x-x_0}. 					\end{equation*}

Se f è derivabile in x_0 allora il valore f'(x_{0}) viene detto derivata di f in x_0.

    \[\quad\]

La derivata f'(x_0) rappresenta il coefficiente angolare della retta tangente al grafico della funzione nel punto (x_0,f(x_0)). Spesso si utilizza il cambio di variabile introdotto in precedenza e si definisce in maniera equivalente

    \[f'(x_0)=\lim_{\Delta x \to 0} \dfrac{\Delta f(x_0)}{\Delta x}= \lim_{\Delta x \to 0} \dfrac{f(x_0+\Delta x)-f(x_0)}{\Delta x}.\]

Graficamente si intuisce che facendo tendere l’incremento \Delta x a zero, il valore f(x_{0}+\Delta x) si avvicina a f(x_{0}); in questo modo la retta secante tende alla retta tangente al grafico di f nel punto (x_{0}, f(x_0)).

Definizione 1.4 (retta tangente). Siano f\colon A\subseteq\mathbb{R}\to \mathbb{R} e x_0\in A un punto di accumulazione per A. Una retta di equazione y=mx+q si dice retta tangente al grafico di f nel suo punto (x_0,f(x_0)) se si ha

(2)   \begin{equation*} 							\lim_{x \to x_0} \dfrac{f(x)-mx-q}{x-x_0}=0. 					\end{equation*}

Inoltre, se f è continua in x=x_0 e il limite del rapporto incrementale esiste me è infinito, diremo che la retta di equazione x=x_0 è retta tangente verticale al grafico di f nel punto (x_0,f(x_0)).

    \[\quad\]

Osservazione 1.5. Si vede facilmente che la retta che soddisfa (2), se esiste, è unica e diremo che essa rappresenta la migliore approssimazione lineare al grafico di f nell’intorno del punto (x_0,f(x_0)). Notiamo che dalla definizione 1.4 si deduce che f è derivabile in x_0 se e solo se esiste la retta tangente al grafico di f nel suo punto (x_0,f(x_0)) e inoltre che questa ha equazione

    \[\boxcolorato{analisi}{{ 			y= f(x_0)+f'(x_0)(x-x_0). 			}}\]

Infatti, sostituendo la (3) in (2) abbiamo

    \[\lim_{x \to x_0} \dfrac{f(x)-(f(x_0)+f'(x_0)(x-x_0))}{x-x_0}=\lim_{x \to x_0}\left(  \dfrac{f(x)-f(x_0)}{x-x_0}- f'(x_0) \right)=0.\]

    \[\quad\]

Definizione 1.6 (funzione derivata prima). Sia A \subseteq \mathbb{R} e sia f\colon A \to \mathbb{R} una funzione. Diciamo che f è derivabile in A se è derivabile in x_0 per ogni x_{0} \in A. In questo caso viene ad essere definita una nuova funzione, la funzione derivata prima

    \[f'\colon A \to \mathbb{R},\]

che assegna ad ogni punto x\in A la derivata f'(x) della funzione f in tale punto.

    \[\quad\]

Le notazioni utilizzate in letteratura per la funzione derivata sono varie; riportiamo di seguito le più comuni:

    \[f^\prime,\quad  \dfrac{df}{dx},\quad \dfrac{d}{dx}f,\quad Df.\]

Quando, più in generale, il punto x_0 nella definizione 1.3 è soltanto un punto di accumulazione destro o sinistro per A, possiamo parlare di derivata destra f_{+}^{\prime}(x_{0}) o di derivata sinistra f_{-}^{\prime}(x_{0}) di f in x_{0}, che corrispondono rispettivamente al limite destro e al limite sinistro del rapporto incrementale.

    \[\quad\]

Definizione 1.7 (derivata destra e sinistra). Sia f\colon A\subseteq\mathbb{R}\to\mathbb{R} una funzione. Supponiamo che x_0 \in A sia un punto di accumulazione a destra per A. Se esiste finito il seguente limite

(3)   \begin{equation*} 			f^\prime_+(x_0)=\lim_{h\to 0^+}\dfrac{f(x_0+h)-f(x_0)}{h}, 				\end{equation*}

allora f si dice derivabile a destra in x_0 e (3) si definisce derivata destra di f in x_0. Analogamente, supponiamo che x_0 \in A sia un punto di accumulazione a sinistra per A. Se esiste finito il seguente limite

(4)   \begin{equation*} 				f^\prime_-(x_0)=\lim_{h\to 0^-}\dfrac{f(x_0+h)-f(x_0)}{h}, 				\end{equation*}

allora f si dice derivabile a sinistra in x_0 (4) si definisce derivata sinistra di f in x_0.

    \[\quad\]

Notiamo che f è derivabile in x_{0} se e solo se f ammette derivata destra e sinistra e queste due coincidono.

Notazione 1.8. Nell’ipotesi in cui i limiti (3) o (4) esistano ma siano infiniti, useremo la notazione f^\prime_+(x_0)=\pm\infty e f^\prime_-(x_0)=\pm\infty.

    \[\quad\]

Definizione 1.9 (funzione di classe C^1). Data una funzione f\colon A\subseteq\mathbb{R}\to\mathbb{R} derivabile in A, se la funzione derivata prima f'\colon A \to \mathbb{R} risulta essere continua in A, si dice che f è di classe C^1 in A e si scrive f\in C^1(A).

    \[\quad\]

Presentiamo di seguito alcuni esempi.

Esempio 1.10. Sia f\colon\mathbb{R}\rightarrow \mathbb{R}, f(x) = x^3+4x+1. Dimostriamo che f è derivabile e determiniamo l’espressione della derivata applicando la definizione.

Sia x \in \mathbb{R}. Applicando la definizione di derivata si ha

    \begin{equation*} \begin{aligned}  \lim_{h \to 0} \dfrac{f(x+h)-f(x)}{h} & = \lim_{h \to 0} \dfrac{(x+h)^3 + 4(x+h)+1 - (x^3+4x+1)}{h} =\\ &= \lim_{h \to 0}  \dfrac{\cancel{x^3}+h^3+3hx^2+3xh^2 + \cancel{4x}+4h+\cancel{1} - \cancel{x^3}-\cancel{4x}-\cancel{1}}{h} = \\ & = \lim_{h \to 0}  \dfrac{h^3+3hx^2+3xh^2 +4h}{h} = \lim_{h \to 0}  \dfrac{\cancel{h}(h^2+3x^2+3xh+4)}{\cancel{h}}  = 3x^2+4. \end{aligned} \end{equation*}

Quindi, \forall x \in \mathbb{R}

    \[\boxcolorato{analisi}{{ 				f^\prime(x) =  3x^2+4. 				}}\]

Esempio 1.11. Sia f\colon\mathbb{R}\rightarrow \mathbb{R}, f(x) = x^2-1. Dimostriamo che f è derivabile e determiniamo l’espressione della derivata applicando la definizione.

Sia x \in \mathbb{R}. Applicando la definizione di derivata si ha

(5)   \begin{equation*} \begin{aligned}  \lim_{h \to 0} \dfrac{f(x+h)-f(x)}{h} & = \lim_{h \to 0} \dfrac{(x+h)^2-1-(x^2-1)}{h} = \\ & = \lim_{h \to 0} \dfrac{\cancel{x^2}+h^2+2hx-\cancel{1}-\cancel{x^2}+\cancel{1}}{h} = \\ & = \lim_{h \to 0} \dfrac{h^2+2hx}{h} = \lim_{h \to 0} \dfrac{\cancel{h}(h+2x)}{\cancel{h}} = 2x. \end{aligned} \end{equation*}

Quindi, \forall x \in \mathbb{R}

    \[\boxcolorato{analisi}{{ 				f^\prime(x) =  2x. 				}}\]

Esempio 1.12. Sia f\colon\mathbb{R}\setminus \{ 0 \}\rightarrow \mathbb{R}, f(x) = \dfrac{1}{x}. Dimostriamo che f è derivabile e determiniamo l’espressione della derivata applicando la definizione.

Sia x \in \mathbb{R}\setminus \{ 0 \}. Applicando la definizione di derivata si ha:

(6)   \begin{equation*} 		\begin{aligned}  			\lim_{h \to 0} \dfrac{f(x+h)-f(x)}{h} & = \lim_{h \to 0} \dfrac{\dfrac{1}{(x+h)}-\dfrac{1}{x}}{h} = \\ 			& = \lim_{h \to 0} \dfrac{\cancel{x}- \cancel{x}-h}{hx(x+h)} = -\frac{1}{x^2}. 		\end{aligned} 	\end{equation*}

Quindi, \forall x \in \mathbb{R}\setminus \{ 0 \}

    \[\boxcolorato{analisi}{{ 						f^\prime(x) = -\frac{1}{x^2}. 						}}\]


Relazioni tra derivabilità e continuità.

Vediamo adesso che relazioni ci sono tra la derivabilità e la continuità di una funzione. Seguendo l’intuito possiamo già capire che, se una funzione non è continua in un punto, allora potrebbe non essere derivabile in quel punto: pensiamo ad esempio ad un punto di discontinuità “di salto”, non sembra possibile parlare di retta tangente in quel punto.

    \[\quad\]

Proposizione 1.13. Sia f\colon A\subseteq\mathbb{R} \rightarrow \mathbb{R} una funzione derivabile in x_{0} \in A. Allora f è continua in x_{0}.

    \[\quad\]

Dimostrazione. Per x \neq x_0 riscriviamo f(x) come segue:

    \begin{equation*} \begin{split} f(x) & = f(x_{0}) \ + \ f(x) - f(x_{0}) \\ & =  f(x_{0}) \ + \underbrace{\frac{f(x) - f(x_{0})}{x-x_{0}}}_\text{rapporto incrementale} (x-x_{0}). \end{split} \end{equation*}

Quindi passando al limite per x \to x_{0} ad entrambi i membri dell’equazione, il rapporto incrementale tende a f'(x_{0}) \in \mathbb{R}, mentre (x-x_{0}) tende a 0 e quindi il loro prodotto tende a 0; dunque f(x) tende a f(x_{0}), che è quello che dovevamo dimostrare.

Viene naturale chiedersi se vale l’implicazione inversa, cioè se una funzione continua in un punto è derivabile in quel punto. La risposta è negativa, e per dimostrare questo fatto proponiamo un controesempio.

Esempio 1.14 (funzione continua e non derivabile in un punto). Consideriamo la funzione f\colon \mathbb{R} \to \mathbb{R}, f(x)=|x| e notiamo che essa è continua su tutto \mathbb{R}. Vogliamo mostrare che f non è derivabile in x_0=0. Infatti, abbiamo

    \[f'_+(0)=\lim_{x \to 0^+} \dfrac{|x|-0}{x-0}=\lim_{x \to 0^+} \dfrac{x}{x}=1,\]

mentre

    \[f'_-(0)=\lim_{x \to 0^-} \dfrac{|x|-0}{x-0}=\lim_{x \to 0^-} \dfrac{-x}{x}=-1.\]

Concludiamo che la funzione è derivabile a destra e a sinistra in x_0=0, ma non è derivabile in x_0=0, poiché i limiti destro e sinistro del rapporto incrementale non coincidono.

Nell’esempio precedente abbiamo visto una funzione che è derivabile a destra e a sinistra, ma che non è derivabile. Esistono anche versioni più “patologiche” di funzioni continue ma non derivabili in un punto. Nel prossimo esempio mostreremo una funzione per cui non esiste nè il limite destro nè il limite sinistro del rapporto incrementale.

Esempio 1.15. Sia f\colon \mathbb{R} \to \mathbb{R} la funzione definita da

    \[ 	f(x) = 	\begin{cases} 	x\sin \left( \dfrac{1}{x} \right),  &\quad\text{se } \ x \neq 0;\\ 	0, &\quad\text{se }  x = 0.\\ 	\end{cases} 	\]

La funzione è continua su \mathbb{R} \setminus \{ 0 \} in quanto composizione di funzioni continue. Inoltre, è continua in 0 perchè \left\vert x\sin \left( \dfrac{1}{x} \right) \right\vert \leq |x| e quindi \lim\limits_{x \to 0} f(x) = 0 = f(0) per il teorema del confronto. Mostriamo adesso che il limite per x \to 0 del rapporto incrementale di f non esiste:

    \begin{equation*} 	\lim_{x \to 0} \dfrac{f(x)-f(0)}{x-0} \ = \ \lim_{x \to 0} \dfrac{x\sin \left( \dfrac{1}{x} \right) - 0}{x} \\ 	= \ \lim_{x \to 0} \sin \left( \dfrac{1}{x} \right). 	\end{equation*}

Si dimostra facilmente che tale limite non esiste e quindi f non è derivabile in 0.

Abbiamo quindi mostrato con questi due esempi e con la proposizione 4.9 che la continuità è una condizione necessaria ma non sufficiente per la derivabilità.

Osservazione 1.16. Supponiamo che ci venga richiesto di determinare se una funzione f sia derivabile in un punto x_0 del proprio dominio. Se notiamo che in tale punto non è continua, possiamo concludere che la funzione non è derivabile in tale punto per la proposizione 4.9.

Osservazione 1.17. La proposizione 4.9 si generalizza facilmente come segue: se f è derivabile da destra o da sinistra in un punto x_0 del suo dominio allora f è continua da destra o da sinistra in x_0. La dimostrazione è analoga ed è lasciata al lettore.


Punti di non derivabilità.

Ipotizziamo che f sia continua in un punto x_0 del proprio dominio ma non derivabile in tale punto. Si possono classificare i punti di non derivabilità nel caso in cui la derivata destra o sinistra esiste finita o infinita:

    \[\quad\]

  • Se f_{+}^{\prime}(x_{0}) \neq f_{-}^{\prime}(x_{0}) e almeno una delle due è finita diciamo allora che x_{0} è un punto angoloso per f.

    Esempio: f(x) = |x-x_0|, x_{0} è un punto angoloso per f.

  •  

  • Se f_{+}^{\prime}(x_{0}) \neq f_{-}^{\prime}(x_{0}) e sono entrambe infinite diciamo allora che x_{0} è un punto di cuspide per f.

    Esempio: f(x) = \sqrt{|x-x_{0}|}, x_{0} è un punto di cuspide per f.

  •  

  • Se f_{+}^{\prime}(x_{0}) = f_{-}^{\prime}(x_{0}) e sono entrambe infinite diciamo allora che x_{0} è un punto a tangente verticale per f.

    Esempio: f(x) = \sqrt[3]{x-x_{0}}, x_{0} è un punto di flesso a tangente verticale per f.

    \[\quad\]

    \[\quad\]

    \[\quad\]

Rendered by QuickLaTeX.com

    \[\quad\]

    \[\quad\]


 
 

Regole di derivazione

Introduzione.

Andiamo adesso ad enunciare e dimostrare i principali teoremi sul calcolo delle derivate.

Teorema 2.1 (proprietà di linearità).

    \[\quad\]

  1. (Somma) Siano f,g\colon A\subseteq\mathbb{R}\to \mathbb{R} due funzioni derivabili in A. Consideriamo la funzione

        \[z\colon A\to \mathbb{R},\;z(x)=f(x)+g(x).\]

    Allora, z è derivabile in A e si ha z^\prime(x)=f^\prime(x)+g^\prime(x) \quad \forall x \in A.

  2.  

  3. (Prodotto per una costante) Sia f\colon A\subseteq\mathbb{R}\to \mathbb{R} una funzione derivabile in A e \alpha\in\mathbb{R}. Consideriamo la funzione z\colon A\subseteq \mathbb{R} \to \mathbb{R},\,z(x)=\alpha f(x). Allora, z è derivabile in A e si ha z^\prime(x)=\alpha f^\prime(x)\quad \forall x \in A.

    \[\quad\]

Dimostrazione.

  1. Applicando la definizione di derivata si ha \forall x \in A

        \begin{equation*} \begin{split} z^\prime(x) &= \ \lim_{h \to 0} \frac{f(x+h) + g(x+h) - f(x) + g(x)}{h}= \\ &= \ \lim_{h \to 0} \frac{f(x+h) - f(x)+ g(x+h) - g(x)}{h}= \\ &= \ \lim_{h \to 0} \left( \frac{f(x+h) - f(x)}{h} + \frac{g(x+h) - g(x)}{h} \right)= \\ &\overset{\clubsuit}{=} \ f'(x) + g'(x), \  \end{split} \end{equation*}

    dove in \clubsuit abbiamo usato il fatto che f e g sono funzioni derivabili in A.

  2.  

  3. Applicando la definizione di derivata si ha \forall x \in A

        \begin{equation*} 	\begin{split} 	z'(x) \ &= \ \lim_{h \to 0} \frac{\alpha f(x+h) - \alpha f(x)}{h}=  	\ \lim_{h \to 0} \alpha\, \frac{f(x+h) - f(x)}{h}= \ \alpha \lim_{h \to 0} \frac{f(x+h) - f(x)}{h}= \ \alpha f'(x). 	\end{split} \end{equation*}

Osservazione 2.2. La dimostrazione che \left(f-g\right)'=f^\prime-g^\prime segue facilmente dal Teorema 2.1.

Teorema 2.3 (regola di Leibniz). Siano f,g\colon A\subseteq\mathbb{R}\to \mathbb{R} due funzioni derivabili in A. Consideriamo la funzione

    \[z\colon A\to \mathbb{R},\,z(x)=f(x)g(x).\]

Allora, z è derivabile in A e si ha z^\prime(x)=f^\prime(x)g(x)+f(x)g^\prime(x)\quad \forall x \in A.

    \[\quad\]

Dimostrazione. Applicando la definizione di derivata si ha \forall x \in A

    \begin{equation*} 	\begin{split} 	z^\prime(x) &= \ \lim_{h \to 0} \frac{f(x+h)g(x+h) - f(x)g(x)}{h} \\ 	&= \ \lim_{h \to 0} \frac{f(x+h)g(x+h) - [f(x)g(x+h) - f(x)g(x+h)] - f(x)g(x)}{h} \\ 	&= \ \lim_{h \to 0} \frac{(f(x+h) - f(x))g(x+h) + f(x)(g(x+h) - g(x))}{h} \\ 	&= \ \lim_{h \to 0} \left( \frac{\big(f(x+h) - f(x)\big)g(x+h)}{h} + \frac{f(x)\big(g(x+h) - g(x)\big)}{h} \right) \\ 	&\overset{\clubsuit}{=} \ f'(x)g(x) + f(x)g'(x), \  	\end{split} 	\end{equation*}

dove in \clubsuit abbiamo usato il fatto che g(x+ h) tende a g(x) per h \to 0, ossia la continuità di g, che segue dalla Proposizione 4.9.

Teorema 2.4 (regola della catena). Siano A, B \subseteq \mathbb{R}, g\colon A \rightarrow B e f\colon B \rightarrow \mathbb{R} due funzioni tali che g sia derivabile in A e f sia derivabile in {\rm Im}(g) \subseteq B. Consideriamo la funzione

    \[z=f\circ g\colon A\to \mathbb{R},\;z(x)=f(g(x)).\]

Allora, z è derivabile in A e si ha z^\prime(x)=f^\prime(g\left(x\right))g^\prime(x)\quad \forall x \in A.

    \[\quad\]

Dimostrazione. Sia x \in A e y=g(x). Supponiamo per il momento che g(x+h)\neq g(x) definitivamente per h \to 0. Applicando la definizione di derivata, abbiamo

(7)   \begin{equation*} 		\begin{split} 			z^\prime(x) &= \lim_{h \to 0} \frac{f\left(g\left(x+h\right)\right) - f\left(g\left(x\right)\right)}{h} = \lim_{h\to 0	}\left(\dfrac{f\left(g\left(x+h\right)\right)-f\left(g\left(x\right)\right)}{g(x+h)-g(x)}\cdot \dfrac{g\left(x+h\right)-g\left(x\right)}{h}\right), 		\end{split} 	\end{equation*}

dove, ponendo

    \[k\coloneqq g(x+h) - y\]

e sfruttando la continuità di g, quando h \to 0 abbiamo che g(x+h) \to y e quindi k \to 0.

Ricordando i teoremi algebrici sui limiti, dall’equazione (7) si ha

    \[z^\prime(x)=\lim_{k\to0}\dfrac{f\left(y+k\right)-f(y)}{k}\cdot \lim_{h\to 0}\dfrac{g(x+h)-g(x)}{h}=f^\prime(y)g^\prime(x)=f(g(x))g^\prime(x).\]

Rimane da studiare il caso in cui per ogni \varepsilon>0 esiste un h_{\varepsilon} \in (-\varepsilon, \varepsilon) tale che g(x+h_{\varepsilon})=g(x). In questo caso non possiamo ragionare come in (7), in quanto abbiamo un problema di divisione per 0. Notiamo però che, poiché g è derivabile in x, si ha

(8)   \begin{equation*} 			g^\prime(x)=\lim_{h \to 0} \frac{g(x+h) - g(x)}{h}= \lim_{\varepsilon \to 0} \frac{g(x+h_{\varepsilon}) - g(x)}{h_{\varepsilon}}=0. 	\end{equation*}

Per concludere la dimostrazione bisogna dunque dimostrare che z è derivabile in x e che vale z'(x)=0.

Poiché f è derivabile per ipotesi in g(x), esistono M,\epsilon>0 tali che

(9)   \begin{equation*} 		\vert f(w)-f(g(x))\vert \leq M \vert w-g(x) \vert \qquad \forall w \in (g(x)-\epsilon, g(x)+ \epsilon). 	\end{equation*}

Infine, per la continuità di g in x, esiste \delta>0 tale che \forall h \in (-\delta,\delta) si ha \vert g(x+h) - g(x) \vert < \epsilon. Possiamo dunque sostituire w=g(x+h) con \vert h \vert < \delta in (9), dividere per |h| e, passando al limite per h \to 0, utilizzare (8) per ottenere

(10)   \begin{equation*} 	\left\vert \frac{	 f(g(x+h))-f(g(x))}{h} \right\vert \leq M 	\left\vert \frac{ g(x+h) - g(x)}{h} \right\vert \to 0, \qquad h \to 0. \end{equation*}

Proposizione 2.5 (regola di derivazione del reciproco). Sia f\colon A\subseteq\mathbb{R}\to \mathbb{R} una funzione derivabile in A e non identicamente nulla. Consideriamo l’insieme B=\{x\in A\colon f(x)\neq0\} e la funzione

    \[z\colon B \to \mathbb{R},\;z(x)=\dfrac{1}{f(x)}.\]

Allora, z è derivabile in B e si ha z^\prime(x)=-\dfrac{f^\prime(x)}{f^2(x)}\quad \forall x \in B.

    \[\quad\]

Dimostrazione. Siccome B è aperto1, ogni suo punto è di accumulazione per B. Per indagare la derivabilità di z, introduciamo la funzione g\colon \mathbb{R} \setminus \{ 0 \} \to \mathbb{R}, \; g(x)= \dfrac{1}{x}. Abbiamo visto nell’esempio 1.12 che g è derivabile. Osserviamo che z=g\circ f, dunque per il teorema 2.4 otteniamo

    \[z^\prime(x)= g^\prime(f(x))\cdot f^\prime (x)=-\dfrac{1}{(f(x))^2} \cdot f^\prime (x)\quad \forall x \in B.\]


  1. Osserviamo che B è la controimmagine dell’aperto \mathbb{R} \setminus \{ 0 \} tramite la funzione f, che è continua per ipostesi, e dunque anche B è aperto.

Proposizione 2.6 (regola di derivazione dei quozienti). Siano f,g\colon A\subseteq\mathbb{R}\to \mathbb{R} due funzioni derivabili in A. Consideriamo l’insieme B=\{x\in A\colon g(x)\neq 0\} e la funzione

    \[z\colon B\to \mathbb{R},\;z(x)=\dfrac{f(x)}{g(x)}.\]

Allora, z è derivabile in B e si ha z^\prime(x)=\dfrac{f^\prime(x)g(x)-f(x)g^\prime(x)}{g^2\left(x\right)}\quad \forall x \in B.

    \[\quad\]

Dimostrazione. Siccome B è aperto, ogni suo punto è di accumulazione per B. Per provare la tesi, usiamo il tereoma 2.3 e la proposizione 2.5. Sia x \in B, abbiamo:

    \begin{equation*} \begin{split} z^\prime(x)=\left( \frac{f(x)}{g(x)} \right)^\prime\ &= \ \left( f(x) \frac{1}{g(x)} \right)^\prime\ = \ f'(x)\frac{1}{g(x)} - f(x) \frac{g'(x)}{g(x)^{2}} =\ \frac{f'(x)g(x) - f(x)g'(x)}{g(x)^{2}} \ . \end{split} \end{equation*}

Il prossimo risultato riguarda la derivabilità della funzione inversa. Prima di enunciarlo, presentiamo un risultato preliminare che non dimostreremo in queste note. La sua dimostrazione si trova in Funzioni continue – teoria, teorema 5.15.

Teorema 2.7 (continuità della funzione inversa). Siano I,J \subseteq \mathbb{R} due intervalli, f\colon I \to J una funzione invertibile e sia x_0 \in I. Supponiamo che f sia continua in x_0. Allora la funzione inversa f^{-1} è continua in y_0 = f(x_0).

Teorema 2.8 (derivata della funzione inversa). Siano I,J \subseteq \mathbb{R} due intervalli, f\colon I \to J una funzione invertibile e sia x_0 \in I. Supponiamo che f sia derivabile in x_0 e che f'(x_0) \ne 0. Allora la funzione inversa f^{-1} è derivabile in y_0 = f(x_0) e vale:

    \[Df^{-1}(y_0) = \frac{1}{f^\prime(f^{-1}(y_0))}.\]

    \[\quad\]

Dimostrazione. Utilizzando il cambio di variabile

    \[x=f^{-1}\left(y\right) \iff y=f(x)\]

si ha

    \begin{equation*} 		\begin{split} 			Df^{-1}\left(y_0\right)=\lim_{y \to y_0} \frac{f^{-1}\left(y\right)-f^{-1}\left(y_0\right)}{y-y_0}  \overset{f^{-1}\left(y\right) = x}{=} \ \lim_{x \to x_0} \frac{x-x_0}{f\left(x\right)-y_0} \ = \frac{1}{f'(x_0)}, 		\end{split} 	\end{equation*}

dove abbiamo usato il fatto che quando y\to y_0 si ha che x=f^{-1}\left(y\right)\to x_0=f^{-1}\left(y_0\right) perché f^{-1} è continua dal teorema 2.7.

Terminiamo questa sezione con due lemmi.

Notazione 2.9. Dato A \subseteq \mathbb{R}, indichiamo con -A l’insieme \{ -x\colon x \in A \}.

    \[\quad\]

Lemma 2.10 (derivata di una funzione pari). Sia A\subseteq\mathbb{R} un insieme simmetrico rispetto all’origine, ovvero tale che A=-A. Se f\colon A\to \mathbb{R} è una funzione pari e derivabile in A, allora f^\prime è dispari.

    \[\quad\]

Dimostrazione. Una funzione pari è una funzione tale che f(x)=f(-x) \forall x \in A. Allora sfruttando il teorema 2.4 si ha f^\prime(x)=-f^\prime(-x) cioè la la tesi. In alternativa, possiamo applicare la definizione di derivata, ottenendo che per x \in A

    \[\begin{aligned} f^\prime(x)&=\lim_{t\to x}\dfrac{f(t)-f(x)}{t-x}\overset{\clubsuit}{=}-\lim_{t\to x}\dfrac{f(-t)-f(-x)}{-t-(-x)}=-f^\prime(-x), \end{aligned}\]

dove in \clubsuit abbiamo sfruttato il fatto che f è una funzione pari.

Lemma 2.11 (derivata di una funzione dispari). Sia A\subseteq\mathbb{R} un insieme simmetrico rispetto all’origine, ovvero tale che A=-A. Se f\colon A\to \mathbb{R} è una funzione dispari e derivabile in A, allora f^\prime è pari.

    \[\quad\]

Dimostrazione. Una funzione dispari è una funzione tale che f(x)=-f(-x) allora sfruttando il teorema 2.4 si ha f^\prime(x)=f^\prime(-x) cioè la tesi. In alternativa, possiamo applicare la definizione di derivata, ottenendo che per x \in A

    \[\begin{aligned} f^\prime(x)=\lim_{t\to x}\dfrac{f(t)-f(x)}{t-x}\overset{\clubsuit}{=}\lim_{t\to x}\dfrac{-f(-t)+f(-x)}{t-x}=\lim_{h\to0}\dfrac{f(-t)-f(-x)}{-t+x}=f^\prime(-x), \end{aligned}\]

dove in \clubsuit abbiamo sfruttato il fatto che f è una funzione dispari.


Derivate delle funzioni elementari.

Vediamo adesso le derivate delle principali funzioni elementari. Una volta apprese quest’ultime, insieme ai teoremi algebrici appena visti, potremo calcolare la maggior parte delle derivate che incontreremo senza dover utilizzare la definizione.

Notazione 2.12. Siccome le notazioni che sono state utilizzate nel corso del tempo sono tante, cercheremo di abituare il lettore a quelle più comuni, come ad esempio

    \[f'(x) = \dfrac{d\,f(x)}{dx}.\]

Quest’ultima risulta essere una tra le prime notazioni utilizzate per indicare la derivata di una funzione (risale infatti al XVIII secolo) ed ha quindi una certa importanza storica. Inoltre, ha il vantaggio di ricordare la definizione stessa di derivata:

    \[\dfrac{d\,f(x)}{dx}=\lim_{\Delta x \to 0} \frac{\Delta f(x)}{\Delta x}, \quad \text{dove }  \quad \Delta f (x)= f(x + \Delta x)-f(x).\]

Esempio 2.13. Sia f\colon\mathbb{R}\to\mathbb{R},\,f(x)=k\in\mathbb{R}. Dimostriamo che f è derivabile in \mathbb{R} e che risulta: \dfrac{d\,k}{dx}=0.

Sia x\in \mathbb{R}. Applicando la definizione di derivata si ha:

    \[\dfrac{d\,f(x)}{dx}= \lim_{\Delta x\to 0} \dfrac{\Delta f(x)}{\Delta x}=\lim_{\Delta x\to 0}\dfrac{k-k}{\Delta x}=0.\]

Esempio 2.14. Sia f\colon\mathbb{R}\to\mathbb{R},\,f(x)=x. Dimostriamo che f è derivabile in \mathbb{R} e che risulta: \dfrac{d\,x}{dx}=1.

Sia x\in \mathbb{R}. Applicando la definizione di derivata si ha:

    \[\begin{aligned} 	&\dfrac{d\,x}{dx}=\lim_{\Delta x\to0}\dfrac{(x+ \Delta x)-x}{\Delta x}=\lim_{\Delta x\to0}\dfrac{\cancel{x}+\Delta x-\cancel{x}}{\Delta x}=1. 	\end{aligned}\]

Notazione 2.15. Utilizzeremo d’ora in avanti la seguente notazione, più moderna e allo stesso tempo più pratica, per indicare la derivata:

    \[f'(x)=Df(x).\]

Inoltre, per indicare l’incremento nel calcolo del limite utilizzeremo la variabile h=\Delta x.

Esempio 2.16. Sia f\colon\mathbb{R}\to\mathbb{R},\,f(x)=\sin x. Dimostriamo che f è derivabile in \mathbb{R} e che risulta

    \[D\sin( x)=\cos x, \quad x \in \mathbb{R}.\]

Sia x\in \mathbb{R}. Applicando la definizione di derivata si ha:

    \[\begin{aligned} D\sin( x)&=\lim_{h\to0}\dfrac{\sin(x+h)-\sin (x)}{h}\overset{\clubsuit}{=}\\ 	&=\lim_{h\to0}\dfrac{\sin (x)\cos (h)+\cos (x)\sin (h)-\sin (x)}{h}=\\ 	&=\lim_{h\to0}\left(\sin (x) \left( \dfrac{\cos (h)-1}{h}\right)+\cos (x)\left( \dfrac{\sin (h)}{h}\right)\right)\overset{\diamondsuit}{=}\cos x. 	\end{aligned}\]

dove in \clubsuit abbiamo usato la regola di somma e sottrazione per il seno e in \diamondsuit abbiamo usato i limiti notevoli.

Esempio 2.17. Sia f\colon\mathbb{R}\to\mathbb{R},\,f(x)=\cos x. Dimostriamo che f è derivabile in \mathbb{R} e che risulta

    \[D\cos(x)=-\sin x, \quad x \in \mathbb{R}.\]

Sia x\in \mathbb{R}. Applicando la definizione si ha:

    \[\begin{aligned} 	D\cos(x)&=\lim_{h\to 0} \dfrac{\cos (x+h)-\cos (x)}{h}\overset{\clubsuit}{=}\\ 	&=\lim_{h\to 0}\dfrac{\cos (x) \cdot \cos (h)-\sin (x)\sin (h)-\cos (x)}{h}=\\   	&=\lim_{h\to 0}\left(\cos x\left(\dfrac{\cos (h)-1}{h}\right)-\sin x\left(\dfrac{\sin h}{h}\right)\right)\overset{\diamondsuit}{=}-\sin x. 	\end{aligned}\]

dove in \clubsuit abbiamo utilizzato la regola di somma e sottrazione del coseno e in \diamondsuit abbiamo applicato i limiti notevoli.

Esempio 2.18. Sia A=\left\{x\in\mathbb{R}\colon\,x\neq \dfrac{\pi}{2}+k\pi,k\in\mathbb{Z}\right\} e f\colon A\to\mathbb{R},\,f(x)=\tan x. Dimostriamo che f è derivabile in A e che

    \[D\tan(x)=\dfrac{1}{\cos^2x}=1+\tan^2x, \quad x \in A.\]

Sia x\in A. Applicando la definizione di derivata si ha:

    \[\begin{aligned} 	D\tan(x)&=\lim_{h\to 0}\dfrac{\tan (x+h)-\tan x}{h}\overset{\clubsuit}{=}\\ 	&=\lim_{h\to 0}\left(\dfrac{\dfrac{\tan x+\tan (h)}{1-\tan x\tan (h)}-\tan x}{h}\right)=\\ 	&=\lim_{h\to 0}\left(\dfrac{\cancel{\tan x}+\tan (h)-\cancel{\tan x}+\tan^2x\cdot \tan (h)}{h(1-\tan x\tan (h))}\right)=\\ 	&=\lim_{h\to 0}\dfrac{\tan (h)}{h}\left(\dfrac{1+\tan^2 x}{1-\tan x\tan (h)}\right)\overset{\diamondsuit}{=}\\ 	&\overset{\diamondsuit}{=} 1+\tan^2 x= \dfrac{1}{\cos^2 x} 	\end{aligned}\]

dove in \clubsuit abbiamo usato la formula di somma e sottrazione per la tangente e in \diamondsuit abbiamo usato i limiti notevoli. Nell’ultima uguaglianza abbiamo utilizzato l’identità fondamentale: \sin^2(x)+ \cos^2(x)=1.

Esempio 2.19. Sia A=\{x\in\mathbb{R}\colon\,x\neq k\pi,k\in\mathbb{Z}\} e f\colon A\to\mathbb{R}, \; f(x)=\cot x. Dimostriamo che f è derivabile in A e che risulta

    \[D\cot x=-\dfrac{1}{\sin^2 x}=-(1+\cot^2x), \quad x \in A.\]

Sia x\in A. Applicando la definizione di derivata si ha:

    \[\begin{aligned} 	D\cot x&=\lim_{h\to 0}\dfrac{\cot \left(x+h\right)-\cot\left(x\right)}{h}\overset{\clubsuit}{=}\\ 	&=\lim_{h\to 0}\dfrac{\dfrac{1}{\tan\left(x+h\right)}-\dfrac{1}{\tan x}}{h}=\lim_{h\to 0}\dfrac{\tan x-\tan \left(x+h\right)}{h\tan x \,\tan \left(x+h\right)}=\\ 	&=\lim_{h\to 0}\left( \dfrac{\tan x -\dfrac{\tan x+\tan h}{1-\tan x \,\tan h }}{h \tan x \tan\left(x+h\right)} \right)=\\ 	&=\lim_{h\to 0}\dfrac{\cancel{\tan x} -\tan^2 x \tan h -\cancel{\tan x}- \tan h}{h\tan x \tan \left(x+h\right)\left(1-\tan x \tan h\right)}=\\ 	&=\lim_{h\to 0}\dfrac{-\tan h \left(1+\tan^2 x\right)}{h\tan x \tan \left(x+h\right)\left(1-\tan x \tan h\right)}=\\ 	&=-\dfrac{1+\tan^2x}{\tan x }\lim_{h\to 0}\left( \dfrac{\tan h}{h}\cdot \dfrac{1}{\tan\left(x+h\right)\left(1-\tan x \tan h\right)} \right)\overset{\diamondsuit}{=}\\ 	&\overset{\diamondsuit}{=}-\dfrac{1+\tan^2 x}{\tan^2 x }=-\left(\dfrac{1 }{\cos^2 x}\right)\left(\dfrac{\cos^2 x}{\sin^2 x }\right)=-\dfrac{1}{\sin^2 x} 	\end{aligned}\]

dove in \clubsuit abbiamo riscritto \cot x=\dfrac{1}{\tan x} e in \diamondsuit abbiamo applicato i limiti notevoli e la continuità della tangente. Era altrimenti possibile utilizzare la proposizione 2.5 e l’esempio 2.18 per ottenere la tesi.

Esempio 2.20. Sia f\colon\mathbb{R}\to\mathbb{R},\,f(x)=a^x\,\,\text{con}\,\,a>0. Dimostriamo che f è derivabile in \mathbb{R} e che risulta

    \[D \,a^x=a^x\ln a,\quad x\in \mathbb{R}.\]

Applicando la definizione di derivata si ha:

    \[\begin{aligned} 	&D\, a^x=\lim_{h\to0}\dfrac{a^{x+h}-a^x}{h}=a^x\lim_{h\to0}\dfrac{a^h-1}{h}\overset{\clubsuit}{=}a^x\ln a, 	\end{aligned}\]

dove in \clubsuit abbiamo applicato i limiti notevoli. Osserviamo che per a=e si ha D\,e^x=e^x.

Esempio 2.21. Sia a>0\,\,\wedge\,\, a\neq 1 e A=(0, +\infty). Consideriamo la funzione f\colon A\to\mathbb{R},\,f(x)=\log_a\left(x\right). Dimostriamo che f è derivabile in A e che risulta

    \[D\log_a x=\dfrac{1}{x\ln a}, \quad x \in A.\]

Sia x\in A. Applicando la definizione di derivata si ha:

    \[\begin{aligned} 	&D\log_a x=\lim_{h\to 0}\dfrac{\log_a(x+h)-\log_a x}{h}\overset{\clubsuit}{=}\lim_{h\to 0}\dfrac{\log_a \left(\dfrac{x+h}{x} \right)}{h}=x\lim_{h\to0}\dfrac{\log_a \left( 1+\dfrac{h}{x}\right)}{\left( \dfrac{h}{x} \right)}\overset{\diamondsuit}{=}\dfrac{1}{x}\log_a e, 	\end{aligned}\]

dove in \clubsuit abbiamo applicato le proprietà dei logaritmi e in \diamondsuit abbiamo applicato i limiti notevoli. Concludiamo osservando che \log_a e=1/\ln a. Notiamo che per a=e si ha D\ln x=\dfrac{1}{x}.

Esempio 2.22. Sia \alpha\in\mathbb{R} e A= (0, +\infty) e f\colon A\to \mathbb{R},\,f(x)=x^\alpha. Dimostriamo che f è derivabile in A e che risulta

    \[D\,x^\alpha=\alpha\, x^{\alpha-1}, \quad x \in A.\]

Riscriviamo la funzione come segue f(x)=e^{\alpha \ln x}. Possiamo dunque utilizzare gli esempi 2.20 e 2.21 e il teorema 2.4 per ottenere che \forall x \in A

    \[D(x^\alpha)= D(e^{\alpha \ln x})=e^{\alpha \ln x}D(\alpha \ln x)=e^{\alpha \ln x}\cdot \dfrac{\alpha}{x}=\dfrac{\alpha}{x}x^\alpha=\alpha x^{\alpha-1}.\]

Osservazione 2.23. Per determinare la derivata di f\colon \mathbb{R} \to \mathbb{R}, \;f(x)=x^n con n\in\mathbb{N} basta considerare l’esempio 2.22 e porre \alpha=n\in\mathbb{N} e si ottiene f^\prime(x)=D\,x^n=nx^{n-1}. Si verifichi che tale formula è valida per ogni x \in \mathbb{R}, e non solo per x>0, calcolando la derivata attraverso la definizione.

Esempio 2.24. Sia n\in\mathbb{N}\setminus \{0\} un numero pari (risp. dispari) e consideriamo la funzione f\colon [0,+\infty) \to \mathbb{R} (risp.f\colon \mathbb{R} \to \mathbb{R}), f(x)=\sqrt[n]{x}. Dimostriamo che f è derivabile in (0, +\infty) (risp. in \mathbb{R}\setminus \{ 0 \}) e che

    \[D(\sqrt[n]{x})= \dfrac{1}{n\sqrt[n]{x^{n-1}}}, \quad  x \in (0, +\infty) \quad (\mbox{risp. }  x \in \mathbb{R}\setminus \{ 0 \}).\]

Se n è pari, utilizziamo il risultato dell’esempio 2.22, ponendo \alpha=\dfrac{1}{n}. Otteniamo

(11)   \begin{equation*} 			D(\sqrt[n]{x})=D\left( x^{\frac{1}{n}} \right)=\frac{1}{n}x^{\frac{1}{n}-1}=\dfrac{1}{n\sqrt[n]{x^{n-1}}}, \quad x>0. 	\end{equation*}

Se n è dispari l’espressione (11) è valida a priori solo per x>0. Osserviamo che in tal caso f è una funzione dispari e dunque la sua derivata è una funzione pari per l’esercizio 2.11 . Dunque, per x<0 otteniamo

    \[f'(x)=f'(-x)=\dfrac{1}{n\sqrt[n]{(-x)^{n-1}}} \overset{\clubsuit}{=} \dfrac{1}{n\sqrt[n]{x^{n-1}}},\]

dove in \clubsuit abbiamo utilizzato il fatto che n-1 è pari. In conclusione, per n dispari

    \[D(\sqrt[n]{x})=\dfrac{1}{n\sqrt[n]{x^{n-1}}}, \quad x\neq 0.\]

Infine, per n \neq 1, f non è derivabile a destra in x=0 (risp. f non è derivabile in x=0), poiché

    \[\lim_{x \to 0^+} \frac{f(x)-f(0)}{x}= 	\lim_{x \to 0^+} x^{\frac{1}{n}-1}= + \infty.\]

Esempio 2.25. Sia f\colon\mathbb{R}\to \left( -\dfrac{\pi}{2},\dfrac{\pi}{2}\right),\;f(x)=\arctan x. Dimostriamo che f è derivabile in \mathbb{R} e che

    \[D\arctan x=\dfrac{1}{1+x^2}, \quad x \in \mathbb{R}.\]

Possiamo applicare il teorema di derivazione della funzione inversa, cf. teorema 2.8. Ricordando l’espressione della derivata della tangente, cf. esempio 2.18, abbiamo che \forall x \in \mathbb{R}

    \[D\arctan x=\dfrac{1}{1+\tan^2 \left(\arctan x\right)}=\dfrac{1}{1+x^2},\]

dove abbiamo sfruttato il fatto che \tan(\arctan x)=x.

Esempio 2.26. Sia f\colon[ -1,1]\to \left[-\dfrac{\pi}{2},\dfrac{\pi}{2}\right],\,f(x)=\arcsin x. Dimostriamo che f è derivabile in (-1,1) e che

    \[D\arcsin x=\dfrac{1}{\sqrt{1-x^2}}, \quad |x|<1.\]

Possiamo applicare il teorema di derivazione della funzione inversa, cf. teorema 2.8. Ricordando l’espressione della derivata della funzione seno, cf. Esempio 2.16, abbiamo che \forall x \in (-1,1)

    \[D\arcsin x=\dfrac{1}{\cos( \arcsin x) }=\dfrac{1}{\sqrt{1-x^2}},\]

dove abbiamo sfruttato il fatto che \cos( \arcsin x)=\sqrt{1-\sin^2(\arcsin x)}=\sqrt{1-x^2}.

Esempio 2.27. Sia f\colon[ -1,1] \to[0,\pi],\,f(x)=\arccos x. Dimostriamo che f è derivabile in (-1,1) e che

    \[D\arccos x=-\dfrac{1}{\sqrt{1-x^2}}, \quad |x| <1.\]

Possiamo applicare il teorema di derivazione della funzione inversa, cf. teorema 2.8. Ricordando l’espressione della derivata della funzione coseno, cf. esempio 2.17, abbiamo che \forall x \in (-1,1)

    \[D\arccos x=-\dfrac{1}{\sin( \arccos x) }=-\dfrac{1}{\sqrt{1-x^2}},\]

dove abbiamo sfruttato il fatto che \sin( \arccos x)=\sqrt{1-\cos^2(\arccos x)}=\sqrt{1-x^2}.

Esempio 2.28. Sia f\colon\mathbb{R}\to \left( -\dfrac{\pi}{2},\dfrac{\pi}{2}\right),\;f(x)={\rm arccot}\,x. Dimostriamo che f è derivabile in \mathbb{R} e che

    \[D\,{\rm arccot}\,x=-\dfrac{1}{1+x^2}, \quad x \in \mathbb{R}.\]

Possiamo applicare il teorema di derivazione della funzione inversa, cf. teorema 2.8. Ricordando l’espressione della derivata della cotangente, cf. esempio 2.19, abbiamo che \forall x \in \mathbb{R}

    \[D\,{\rm arccot}\,x=-\dfrac{1}{1+\cot^2 \left({\rm arccot}\,x\right)}=-\dfrac{1}{1+x^2},\]

dove abbiamo sfruttato il fatto che \cot({\rm arccot}\,x)=x.

Esempio 2.29. Sia f\colon\mathbb{R}\to \mathbb{R},\,f(x)=\sinh x=\dfrac{e^x-e^{-x}}{2} la funzione seno iperbolico. Dimostriamo che f è derivabile in \mathbb{R} e che risulta

    \[D\sinh x=\cosh x, \quad x \in \mathbb{R}.\]

Per ogni x \in \mathbb{R} si ha

    \[\begin{aligned} D\sinh x&=\lim_{h\to 0}\dfrac{\sinh\left(x+h\right)-\sinh x}{h}\overset{\clubsuit}{=}\lim_{h\to 0}\dfrac{\sinh x \cosh h+\cosh x \sinh h-\sinh x}{h}=\\ &=\lim_{h\to 0}\sinh x\left(\dfrac{\cosh x-1}{h}\right)+\cosh x\lim_{h\to 0}\dfrac{\sinh x}{h}\overset{\diamondsuit}{=}\cosh x, \end{aligned}\]

dove in \clubsuit abbiamo applicato la formula di somma e sottrazione per il seno iperbolico e in \diamondsuit abbiamo applicato i limiti notevoli così ottenendo la tesi. Era altrimenti possibile utilizzare l’espressione esplicita

    \[\sinh x=\dfrac{e^x-e^{-x}}{2},\]

e derivare quest’ultima per ottenere la tesi.

Osservazione 2.30. Le derivate del coseno iperbolico, tangente iperbolica e cotangente iperbolica si possono determinare procedendo in modo analogo all’esempio 2.29. Riportiamo nella tabella 2.2 l’espressione di tali derivate.

Esempio 2.31. Sia f\colon\mathbb{R}\to\mathbb{R},\,f(x)={\rm arsinh}(x) la funzione inversa del seno iperbolico. Dimostriamo che f è derivabile in \mathbb{R} e che risulta

    \[D \,{\displaystyle \operatorname {arsinh} (x)}=\dfrac{1}{\sqrt{1+x^2}}, \quad x \in \mathbb{R}.\]

Possiamo utilizzare il risultato dell’esempio 2.29 e il teorema 2.8. Abbiamo che per ogni x \in \mathbb{R}

    \[D{\displaystyle \operatorname {arsinh} (x)}=\dfrac{1}{\cosh\left(\text{arsinh}x \right)}\overset{\clubsuit}{=}\dfrac{1}{\sqrt{1+\sinh^2(\text{arsinh}(x))}}=\dfrac{1}{\sqrt{1+x^2}},\]

dove in \clubsuit abbiamo usato l’identità fondamentale \cosh^2 x -\sinh^2 x=1. Era altrimenti possibile ricordare l’espressione esplicita \text{arsinh}x=\ln\left(1+\sqrt{x^2+1}\right) e derivare quest’ultima per ottenere la tesi.

Osservazione 2.32. Le derivate delle funzioni inverse del coseno iperbolico, tangente iperbolica e cotangente iperbolica si determinano procedendo in modo analogo all’esempio 2.31. Riportiamo nella tabella 2.2 l’espressione di tali derivate.


Tabella delle derivate delle funzioni elementari.

Derivate notevoli

(12)   \begin{equation*}     & f(x)=x^{\alpha} \quad (\alpha \in \mathbb{R})& \quad \quad \quad \quad &f'(x)=\alpha\,x^{\alpha-1}\\[0.2em] \end{equation*}

(13)   \begin{equation*} 				& f(x)=a^x \quad (a >0)& \quad \quad \quad \quad &f'(x)=a^x\ln(a)\\[0.2em] \end{equation*}

(14)   \begin{equation*} 				& f(x)=\log_a(x) \quad (a >0)& \quad \quad \quad \quad &f'(x)=\dfrac{1} {\ln(a)x}\\[0.2em] \end{equation*}

(15)   \begin{equation*} 				& f(x)=\sin(x) & \quad \quad \quad \quad &f'(x)=\cos(x)\\[0.2em] \end{equation*}

(16)   \begin{equation*} 				& f(x)=\cos(x) & \quad \quad \quad \quad &f'(x)=-\sin(x)\\[0.2em] \end{equation*}

(17)   \begin{equation*} 				& f(x)=\tan(x) & \quad \quad \quad \quad &f'(x)=\dfrac{1}{\cos^2(x)}\\[0.2em] \end{equation*}

(18)   \begin{equation*} 				& f(x)=\cot(x) & \quad \quad \quad \quad &f'(x)=-\dfrac{1}{\sin^2(x)}\\[0.2em] \end{equation*}

(19)   \begin{equation*} 				& f(x)=\arcsin(x) & \quad \quad \quad \quad &f'(x)=\frac{1}{\sqrt{1-x^2}}\\[0.2em] \end{equation*}

(20)   \begin{equation*} 				& f(x)=\arccos(x) & \quad \quad \quad \quad &f'(x)=-\frac{1}{\sqrt{1-x^2}}\\[0.2em] \end{equation*}

(21)   \begin{equation*} 				& f(x)=\arctan(x) & \quad \quad \quad \quad &f'(x)=\frac{1}{1+x^2}\\[0.2em] \end{equation*}

(22)   \begin{equation*} 				& f(x)={\rm arccot}(x) & \quad \quad \quad \quad &f'(x)=-\frac{1}{1+x^2}\\[0.2em] \end{equation*}

(23)   \begin{equation*} 				& f(x)=\sinh(x) & \quad \quad \quad \quad &f'(x)=\cosh(x)\\[0.2em] \end{equation*}

(24)   \begin{equation*} 				& f(x)=\cosh(x) & \quad \quad \quad \quad &f'(x)=\sinh(x)\\[0.2em] \end{equation*}

(25)   \begin{equation*} 				& f(x)=\tanh(x) & \quad \quad \quad \quad &f'(x)=\dfrac{1}{\cosh^2(x)}\\[0.2em] \end{equation*}

(26)   \begin{equation*} 				& f(x)={\rm arsinh}(x) & \quad \quad \quad \quad &f'(x)=\frac{1}{\sqrt{x^2+1}}\\[0.2em] \end{equation*}

(27)   \begin{equation*} 				& f(x)={\rm arcosh}(x) & \quad \quad \quad \quad &f'(x)=\frac{1}{\sqrt{x^2-1}}\\[0.2em] \end{equation*}

(28)   \begin{equation*} 				& f(x)={\rm artanh}(x) & \quad \quad \quad \quad &f'(x)=\frac{1}{1-x^2}\\[0.2em] \end{equation*}

    \[\quad\]

    \[\quad\]

Per completezza, ricordiamo che dalla tabella precedente si deducono le seguenti derivate:

    \[\quad\]

    \[\quad\]

(29)   \begin{equation*} & g(x)=\sqrt{x} & \quad \quad \quad \quad &g'(x)=\dfrac{1}{2\sqrt{x}}\\[0.4em] \end{equation*}

(30)   \begin{equation*} 					& f(x)=\sqrt[n]{x} & \quad \quad \quad \quad &f'(x)=\dfrac{1}{n}x^{\frac 1 n -1}\\[0.2em] \end{equation*}

(31)   \begin{equation*} 					& f(x)=e^x & \quad \quad \quad \quad &f'(x)=e^x\\[0.2em] \end{equation*}

(32)   \begin{equation*} 					& f(x)=\ln(x) & \quad \quad \quad \quad &f'(x)=\dfrac{1}{x}\\[0.2em] \end{equation*}

    \[\quad\]

    \[\quad\]


Tabella delle derivate di funzioni composte.

Per comodità del lettore, riportiamo di seguito una lista (non esaustiva) delle derivate di funzioni ottenute componendo le funzioni elementari a funzioni generiche. La tabella si ottiene dalla tabella 2.2 applicando il teorema 2.4.

    \[\quad\]

Derivate di funzioni composte

    \[\quad\]

(33)   \begin{equation*} & g(x)=\left(f(x)\right)^{\alpha} \quad (\alpha \in \mathbb{R}) & \quad \quad \quad \quad &g'(x)=\alpha\,\left(f(x)\right)^{\alpha-1}\cdot f'(x)\\[0.4em] \end{equation*}

(34)   \begin{equation*} 				& g(x)=a^{f(x)}  \quad (a >0)& \quad \quad \quad \quad &g'(x)=f'(x)\cdot a^{f(x)}\ln(a)\\[0.4em] \end{equation*}

(35)   \begin{equation*} 				& g(x)=\log_a(f(x))  \quad (a >0)& \quad \quad \quad \quad  &g'(x)=\dfrac{f'(x)}{\ln(a)f(x)}\\[0.4em] \end{equation*}

(36)   \begin{equation*} 				& g(x)=\sin(f(x)) & \quad \quad \quad \quad &g'(x)=\cos(f(x))\cdot f'(x)\\[0.4em] \end{equation*}

(37)   \begin{equation*} 				& g(x)=\cos(f(x)) & \quad \quad \quad \quad &g'(x)=-\sin(f(x))\cdot f'(x)\\[0.4em] \end{equation*}

(38)   \begin{equation*} 				& g(x)=\tan(f(x)) & \quad \quad \quad \quad &g'(x)=\dfrac{f'(x)}{\cos^2(f(x))}\\[0.4em] \end{equation*}

(39)   \begin{equation*} 				& g(x)=\cot(f(x)) & \quad \quad \quad \quad &g'(x)=-\dfrac{f'(x)}{\sin^2(f(x))}\\[0.4em] \end{equation*}

(40)   \begin{equation*} 				& g(x)=\arcsin(f(x)) & \quad \quad \quad \quad &g'(x)=\frac{f'(x)}{\sqrt{1-\left(f(x)\right)^2}}\\[0.4em] \end{equation*}

(41)   \begin{equation*} 				& g(x)=\arccos(f(x)) & \quad \quad \quad \quad &g'(x)=-\frac{f'(x)}{\sqrt{1-\left(f(x)\right)^2}}\\[0.4em] \end{equation*}

(42)   \begin{equation*} 				& g(x)=\arctan(f(x)) & \quad \quad \quad \quad &g'(x)=\frac{f'(x)}{1+\left(f(x)\right)^2}. \end{equation*}

    \[\quad\]

    \[\quad\]

Possiamo dedurre da queste le seguenti derivate:

    \[\quad\]

    \[\quad\]

(43)   \begin{equation*} & g(x)=\sqrt{f(x)} & \quad \quad \quad \quad &g'(x)=\dfrac{f'(x)}{2\sqrt{f(x)}}\\[0.4em] \end{equation*}

(44)   \begin{equation*} 					& g(x)=\sqrt[n]{f(x)} & \quad \quad \quad \quad &g'(x)=\dfrac{f'(x)}{n\sqrt[ n]{(f(x))^{ n -1}}}\\[0.4em] \end{equation*}

(45)   \begin{equation*} 					& g(x)=e^{f(x)} & \quad \quad \quad \quad &g'(x)=e^{f(x)}\cdot f'(x)\\[0.4em] \end{equation*}

(46)   \begin{equation*} 					& g(x)=\ln(f(x)) & \quad \quad \quad \quad &g'(x)=\dfrac{f'(x)}{f(x)} \end{equation*}

    \[\quad\]

    \[\quad\]


Derivazione del modulo.

Poniamo una particolare attenzione alla derivata della “funzione modulo”, f\colon \mathbb{R} \to 	\mathbb{R}, \; f(x)= |x|, definita come segue: \forall \, x \in \mathbb{R}

    \[\vert x\vert=\max\{-x,x\}= \begin{cases} 	x, &  \text{se $x\geq0$}; \\ 	-x, &  \text{se $x<0$}. \end{cases}\]

Ragionando come nell’esempio 1.14, possiamo ricavare facilmente che tale funzione è derivabile in \mathbb{R}\setminus \{ 0 \} e la sua derivata, f'\colon \mathbb{R}\setminus \{ 0 \} \to \mathbb{R}, vale

(47)   \begin{equation*} 	f'(x)={\rm sgn}(x)=\frac{x}{|x|}=\begin{cases} 		1, \mbox{ se } x >0;\\ 		-1,  \mbox{ se } x <0. 	\end{cases} \end{equation*}

detta funzione “segno”.

Più in generale, data una funzione f\colon A \subseteq \mathbb{R} \to \mathbb{R}, il modulo di f è dato da |f|\colon A \subseteq \mathbb{R} \to \mathbb{R}, definito come segue:

    \[\vert f\vert(x)\coloneqq\vert f(x)\vert= \begin{cases} 	f(x), &  \text{se $f(x)\geq0$}; \\ 	-f(x), &  \text{se $f(x)<0$}. \end{cases}\]

Nella seguente proposizione troviamo una formula per il calcolo della derivata del modulo di una funzione.

Proposizione 2.33 (derivata del modulo di una funzione). Sia f\colon A \subseteq \mathbb{R} \to \mathbb{R} una funzione derivabile in x_0 \in A, e supponiamo che f(x_0) \neq 0.

Allora, la funzione |f|\colon A \subseteq \mathbb{R} \to \mathbb{R} è derivabile in x_0 e vale che

(48)   \begin{equation*} 						\dfrac{d\left \vert f\right \vert}{dx}(x_0)= 						\begin{cases} 							f'(x_0), &  \text{se} \,\,f(x_0)>0;\\\\ 							-f'(x_0), &  \text{se} \,\,f(x_0)<0, 						\end{cases} 					\end{equation*}

ovvero,

(49)   \begin{equation*} 						\dfrac{d\left \vert f\right \vert}{dx}(x_0)={\rm sgn}	(f(x_0)) f^\prime(x_0), 					\end{equation*}

dove {\rm sgn} è la funzione “segno”, cf. (47).

    \[\quad\]

Dimostrazione. La proposizione segue immediatamente dalla “regola della catena”, cf. teorema 2.4.

Osservazione 2.34. La funzione modulo può introdurre punti di non derivabilità: nei valori in cui f cambia segno. Infatti, vedremo in seguito, cfr. esempio 2.14 , che la funzione f(x)=x è derivabile in \mathbb{R}, mentre abbiamo visto nell’esempio 1.14 che la funzione g(x)=|f(x)|=|x| non è derivabile in x_0=0.

Non sempre il modulo di una funzione presenta punti di non derivabilità nei punti in cui f cambia segno. Prendiamo ad esempio la funzione f(x)=\vert x^3\vert. Applicando la proposizione 2.33, otteniamo

    \[f'(x)= 	\begin{cases} 		3x^2, &  \text{se $x>0$} \\ 		-3x^2, &  \text{se $x<0$}. 	\end{cases}\]

Indaghiamo ora la derivabilità in x=0 con un calcolo esplicito, poiché in questo caso non possiamo applicare la proposizione 2.33:

    \[\lim_{x\to0}\dfrac{|x^3|}{x}=\lim_{x\to0}{x|x|}=0.\]

Dunque, la funzione è derivabile anche in x=0, nonostante sia presente il modulo.


 
 

Principali proprietà

Introduzione.

Vediamo ora dei risultati che legano le proprietà della derivata di una funzione con le proprietà della funzione stessa.

Legami tra segno della derivata e monotonia.

Lemma 3.1. Siano f\colon  A \subseteq \mathbb{R} \rightarrow \mathbb{R} e x_0 \in A. Supponiamo che f sia derivabile in x_{0} e che f^{\prime}(x_{0})>0. Allora esiste \delta > 0 per cui:

    \[\quad\]

  • (50)   \begin{equation*}  f(x) > f(x_{0}) \qquad \forall x \in (x_{0}, \ x_{0}+ \delta)\cap A; \end{equation*}

  •  

  • (51)   \begin{equation*} f(x) < f(x_{0}) \qquad \forall x \in (x_{0} - \delta, \ x_{0})\cap A. \end{equation*}

    \[\quad\]

Dimostrazione. Per ipotesi si ha

    \[f^{\prime}(x_{0}) \ = \ \lim_{x \to x_0} \frac{f(x)-f(x_{0})}{x-x_0} \ > \ 0.\]

Per il teorema della permanenza del segno esiste un \delta > 0 tale che si abbia

(52)   \begin{equation*} 	\frac{f(x)-f(x_{0})}{x-x_0} \ > \ 0 \qquad \forall \ x \in (x_{0}-\delta, \ x_{0}+ \delta)\cap A  . \end{equation*}

Possiamo quindi concludere che:

    \[\quad\]

  • Per x \in (x_{0}, \ x_{0}+ \delta)\cap A il denominatore di (52) è positivo e quindi, essendo la frazione positiva, anche il numeratore è positivo. Otteniamo dunque (50).
  •  

  • Per x \in (x_{0}-\delta, \ x_{0})\cap A il denominatore di (52) è negativo e quindi, essendo la frazione positiva, anche il numeratore è negativo. Otteniamo dunque (51).

Il prossimo risultato è una versione globale del lemma precedente.

Lemma 3.2. Sia f\colon (a,b) \rightarrow \mathbb{R} una funzione derivabile. Se f'(x) > 0 \; \forall x \in (a, \ b), allora f è strettamente crescente in (a, b).

    \[\quad\]

Dimostrazione. Siano x,y \in (a,b) tali che x<y. Allora, per il teorema di Lagrange, esiste \xi \in (x,y) tale che

    \[\dfrac{f(y)-f(x)}{y-x}=f'(\xi)>0\quad  \Rightarrow \quad  f(x) < f(y).\]

Osservazione 3.3. Notiamo che, se nel lemma 3.1 supponiamo f^{\prime}(x_{0}) < 0, otteniamo

    \[f(x) < f(x_{0}) \quad \forall \ x \in (x_{0}, \ x_{0}+ \delta)\quad  \mbox{e} \quad f(x) > f(x_{0}) \quad \forall \ x \in (x_{0} - \delta, \ x_{0}).\]

Analogamente, se nel lemma 3.1 supponiamo f'< 0 in tutto un intervallo (a, b) otteniamo che f è strettamente decrescente in (a, b).

Diamo una definizione utile prima del prossimo teorema.

    \[\quad\]

Definizione 3.4. Siano f\colon A \subseteq \mathbb{R}\rightarrow \mathbb{R} e x_0 \in A. Si dice che x_0 è un massimo locale (o relativo) per f se esiste \delta >0 tale che

    \[f(x) \leq f(x_0) \quad \forall\, x \in (x_0-\delta,x_0+\delta) \cap A.\]

Analogamente, si dice che x_0 è un minimo locale (o relativo) per f se esiste \delta >0 tale che

    \[f(x) \geq f(x_0) \quad \forall\, x \in (x_0-\delta,x_0+\delta) \cap A.\]

    \[\quad\]

Utilizzeremo ora il lemma 3.1 per dimostrare un altro risultato molto utile.

    \[\quad\]

Teorema 3.5 (Fermat). Siano f\colon A  \subseteq \mathbb{R} \rightarrow \mathbb{R} e x_0 \in A. Supponiamo che f sia derivabile in x_{0} e che x_{0} sia un punto di minimo (o massimo) locale per f. Allora f^{\prime}(x_{0}) = 0.

    \[\quad\]

Dimostrazione. Per ipotesi x_{0} è un punto di minimo locale per f, cioè esiste un intorno I di x_{0} tale che f(x_{0}) \leq f(x) \ \forall \ x \in I \cap A. Supponiamo per assurdo che f^{\prime}(x_{0}) \neq 0. Allora:

    \[\quad\]

  • Se f^{\prime}(x_{0}) > 0 allora per il lemma 3.1 esiste \delta > 0 tale che f(x) < f(x_{0}) \ \forall \ x \in  I \cap(x_{0} - \delta, \ x_{0}) \cap A.
  •  

  • Se f^{\prime}(x_{0}) < 0 allora per il lemma 3.1 esiste \delta > 0 tale che f(x) < f(x_{0}) \ \forall \ x \in  I \cap(x_{0}, \ x_{0}+\delta)\cap A.

In entrambi i casi si giunge ad un assurdo perché stiamo andando contro l’ipotesi che x_{0} sia un punto di minimo locale. Dunque, si ha necessariamente f'(x_0)=0. La dimostrazione nel caso in cui x_{0} è un punto di massimo locale è analoga.


Derivate destra e sinistra e limiti della derivata.

Vediamo adesso un teorema che mette in relazione la derivabilità di una funzione in un punto con il limite della derivata in quel punto.

Teorema 3.6. Siano f\colon A \subset \mathbb{R} \rightarrow \mathbb{R} e x_{0} \in A un punto di accumulazione destro per A. Supponiamo che f sia continua in un intorno destro I^+ di x_{0} e derivabile in I^+ \backslash \{ x_{0} \}. Se esiste

    \[L = \lim_{x\to x_{0}^{+}} f^{\prime}(x),\]

allora si hanno due possibilità:

    \[\quad\]

  • L = \pm \infty. In questo caso f non è derivabile a destra in x_{0};
  •  

  • L \in \mathbb{R}. In questo caso f è derivabile a destra in x_0 e L = f_{+}^{\prime}(x_{0}).

Un analogo risultato vale per la derivata sinistra.

    \[\quad\]

Dimostrazione. Possiamo assumere senza perdita di generalità che x_0=0 (altrimenti operiamo il cambiamento di variabile x=x_0+y). Per dimostrare il teorema ci avvaliamo del teorema di Lagrange (anche noto come teorema del valor medio).

Sappiamo che f è continua nell’intervallo [0,h], per un certo h >0, e inoltre sappiamo che f è derivabile in (0,h). Possiamo dunque applicare il teorema di Lagrange e concludere che, preso un qualunque x\in (0,h), esiste un certo \xi_x \in (0,x) per cui vale la seguente identità:

    \[\dfrac{f(x)-f(0)}{x}=f^{\prime}(\xi_x).\]

Passando al limite per x \to 0^+, otteniamo

    \[\lim_{x\to 0^+}\dfrac{f(x)-f(0)}{x}= \lim_{x\to 0^+}f^{\prime}(\xi_x).\]

Siccome vale che 0<\xi_x<x, avremo che, per confronto, \xi_x\to 0 per x \to 0^+. Dunque,

    \[\lim_{x\to 0^+}f^{\prime}(\xi_x)=\lim_{x\to 0^+}f^{\prime}(x)=L.\]

Otteniamo dunque che

    \[\lim_{x\to 0^+}\dfrac{f(x)-f(0)}{x}=L,\]

ovvero f è derivabile a destra di 0 e vale f^{\prime}_+(0)=L. La dimostrazione è identica nel caso della derivata sinistra e del limite da sinistra della derivata.

Una conseguenza immediata di questo teorema è il seguente corollario:

Corollario 3.7. Siano f\colon A \subset \mathbb{R} \rightarrow \mathbb{R} e x_{0} \in A un punto di accumulazione per A. Supponiamo che f sia continua in un intorno I di x_{0} e derivabile in I \setminus \{ x_{0} \}. Se esistono finiti entrambi i limiti:

    \[L_{1} = \lim_{x\to x_{0}^{+}} f^{\prime}(x) \quad e \quad L_{2} = \lim_{x\to x_{0}^{-}} f^{\prime}(x),\]

e inoltre L_{1} = L_{2}, allora f è derivabile in x_{0} e f^{\prime}(x_{0}) = L_{1} = L_{2}.

    \[\quad\]

Osservazione 3.8. Notiamo che il corollario 3.7 si può riformulare affermando che la derivata di una funzione non può avere discontinuità eliminabili.

I teoremi precedenti non si possono invertire: il prossimo esempio mostra una funzione derivabile in x_0=0 e tale che i limiti destro e sinistro delle derivate non esistono.

Esempio 3.9. Sia f\colon \mathbb{R} \rightarrow \mathbb{R} la funzione definita da

Osserviamo che f è continua su tutto \mathbb{R}. Infatti, è continua in x \neq 0 in quanto composizione di funzioni continue. Infine, per x \to 0 si ha che x^2 \to 0, mentre \sin{\left (\dfrac{1}{x} \right)} è limitata, dunque concludiamo per il teorema del confronto che

    \[\lim\limits_{x \to 0} x^{2}\sin{\left (\frac{1}{x} \right)} = 0.\]

Si può calcolare la derivata di x^{2}\sin{\left (\dfrac{1}{x} \right)} con i teoremi algebrici, in particolare con la regola di Leibniz, cf. teorema 2.3 e la regola della catena, cf. teorema 2.4. Otteniamo

    \[f'(x)=2x \sin{\left (\frac{1}{x} \right)} - \cos{\left (\frac{1}{x} \right)}, \quad x \neq 0 .\]

Notiamo subito che il limite per x \to 0 di quest’ultima non esiste, nè da destra nè da sinistra. Infatti, il primo termine tende a 0, mentre il secondo, cioè \cos{\left (\dfrac{1}{x} \right)}, non ha limite. Se il limite della somma di questi due termini esistesse, esisterebbe anche il limite di \cos(\dfrac 1 x) per i teoremi algebrici sui limiti, il che è falso. Ciò, però, non implica che f non sia derivabile in x_{0} = 0. Applicando la definizione, abbiamo:

(53)   \begin{equation*} 	\begin{split} 	f'(0)= \ \lim_{x \to 0} \frac{f(x)-f(0)}{x}  = \ &\lim_{x \to 0} \frac{x^{2}\sin{\left (\frac{1}{x} \right)}-0}{x}=\\ 	= \ & \ \lim_{x\to 0} \left[ x \cdot \sin{\left (\frac{1}{x} \right)} \right] \ \overset{\clubsuit}{=} \ 0, 	\end{split} 	\end{equation*}

dove in \clubsuit abbiamo utilizzato il teorema del confronto, cf. esempio 1.15 . In conclusione, f è derivabile in 0 ma ciò non è verificabile analizzando il limite della sua derivata.


 
 

Derivate di ordine superiore e differenziabilità

Derivate di ordine superiore.

Dato A \subseteq \mathbb{R} e una funzione f derivabile in A, ha senso chiedersi se la sua derivata è a sua volta una funzione derivabile in x_0 \in A. In tal caso, diciamo che f è derivabile 2 volte in x_0 e questo limite si chiama derivata seconda di f in x_{0}. Inoltre, se f è derivabile 2 volte in ogni x_0\in A, otteniamo una nuova funzione

    \[f''\colon A \to \mathbb{R},\]

che viene detta derivata seconda di f. Più in generale, ponendo la derivata 0-esima di f, f^{(0)}=f, la derivata prima f^{(1)}=f', possiamo definire la derivata k-esima di f, per k>1, in modo induttivo.

    \[\quad\]

Definizione 4.1 (derivate successive). Siano k>0 un numero naturale, A \subset \mathbb{R} e f\colon A \rightarrow \mathbb{R} una funzione. Supponiamo che f sia derivabile k volte in A e denotiamo con f^{(k)} la derivata k-esima di f. La derivata (k+1)-esima di f, se esiste, è la derivata di f^{(k)} e viene denotata con f^{(k+1)}.

    \[\quad\]

Notiamo che, se f è derivabile k volte in A, allora le sue derivate fino alla (k-1)-esima sono continue in A per la proposizione 4.9. Risulta comodo introdurre una terminologia nel caso in cui anche la funzione f^{(k)} sia continua in A.

    \[\quad\]

Definizione 4.2. Siano k \in \mathbb{N}, A \subseteq  \mathbb{R} e f\colon A \rightarrow \mathbb{R}. Si dice che f \in C^{k}(A) se f è derivabile k volte con continuità in A, cioè tutte le sue derivate fino alla k-esima (inclusa) sono continue in A.

Diremo che f \in C^{\infty}(A) se f \in C^{k}(A) \; \forall \, k \in \mathbb{N}.

    \[\quad\]

Derivata seconda e convessità

Abbiamo visto, nelle discussioni fatte nelle sezioni precedenti, che lo studio della derivata prima di una funzione fornisce informazioni sulla monotonia della stessa e, in particolare, sui suoi estremi relativi.

In questa sezione vediamo come, dallo studio della derivata seconda di una funzione f, si possano ricavare informazioni sulla convessità di f. Ricordiamo la definizione di convessità per comodità del lettore. Per uno studio approfondito di tale concetto si rimanda in funzioni convesse.

    \[\quad\]

Definizione 4.3. Sia I\subseteq \mathbb{R} un intervallo. Una funzione f\colon I \to \mathbb{R} si dice convessa in I se per ogni x_0,x_1 \in I e per ogni t \in (0,1) si ha che

(54)   \begin{equation*} 				f\big((1-t)x_0 + tx_1\big) \leq (1-t)f(x_0)+ tf(x_1). 			\end{equation*}

In altri termini, f è convessa se l’immagine tramite f di una combinazione lineare convessa di x_0,x_1 è dominata dalla corrispondente combinazione lineare convessa delle immagini tramite f di x_0,x_1.

Diciamo inoltre che f è concava se la funzione -f è convessa.

    \[\quad\]

Osservazione 4.4. In termini geometrici, una funzione convessa è una funzione tale per cui il suo epigrafico2 è un insieme convesso3.

    \[\quad\]


  1. Ricordiamo che l’epigrafico di una funzione f\colon A \subseteq \mathbb{R} \to \mathbb{R} è l’insieme \{ (x,y) \in A\times \mathbb{R}\colon y \geq f(x) \}.

  1. Un insieme U \subseteq \mathbb{R}^n si dice convesso se il segmento che congiunge due punti qualunque di U è interamente contenuto in U.

    \[\quad\]

Il principale risultato che lega la convessità alla derivata seconda può essere enunciato come segue.

Teorema 4.5. Sia I \subset \mathbb{R} un intervallo e sia f\colon I \rightarrow \mathbb{R} derivabile due volte in I. Allora:

    \[\quad\]

  • f è convessa in I \ \iff \ f^{\prime \prime}(x) \geq 0 \forall \, x \in I;
  •  

  • f è concava in I \ \ \iff \ f^{\prime \prime}(x) \leq 0 \forall \, x \in I.

    \[\quad\]

Per dimostrare questo teorema ci serviamo di due lemmi. Il primo stabilisce una proprietà fondamentale delle funzioni convesse e ne riportiamo solamente l’enunciato, mentre il secondo è un lemma specifico che riguarda le derivate e ne riportiamo quindi la dimostrazione:

Lemma 4.6. Sia I \subset \mathbb{R} un intervallo e sia f\colon I \rightarrow \mathbb{R}. Dati x,y \in I tali che x \neq y, poniamo

    \[r_{f}(x,y)\coloneqq\frac{f(y)-f(x)}{y-x} .\]

Allora valgono le seguenti:

    \[\quad\]

    \[\quad\]

  1.     \[f \text{ convessa in } I \iff \forall \ a,b,c \in I \; \mbox{ tale che } \; a<b<c,  \mbox{ si ha } \ r_f(a,b)\leq r_f(a,c)\leq r_f(b,c);\]

  2.  

  3.     \[f \text{ convessa in } I \iff \ \forall \ a,b,c \in I  \ \mbox{ tale che } \ a<b<c,  \mbox{ si ha } \  r_f(a,b)\leq r_f(b,c).\]

    \[\quad\]

Lemma 4.7. Sia I \subset \mathbb{R} un intervallo e sia f\colon I \rightarrow \mathbb{R} derivabile in I. Allora:

    \[f \text{ è convessa in } I \iff f^{\prime} \text{ è debolmente crescente in } I .\]

    \[\quad\]

Dimostrazione.\Rightarrow” Vogliamo mostrare che, i dati x e y in I, x < y \Rightarrow f^{\prime}(x)\leq f^{\prime}(y). Per ipotesi f è convessa, e possiamo applicare il lemma 4.6.

Siccome x < y, per ogni h \in (0, y-x) vale x<x+h<y, quindi per il punto 1) del lemma 4.6:

    \[\frac{f(x+h)-f(x)}{h} \ \leq \ \frac{f(y)-f(x)}{y-x},\]

e passando al limite per h\to 0^{+} otteniamo:

(55)   \begin{equation*}  	f^{\prime}(x) \ \leq \ \frac{f(y)-f(x)}{y-x} \; . 	\end{equation*}

Per i suddetti valori di h, vale anche x<y-h<y, quindi sempre per il punto 1) del lemma 4.6 si ha:

    \[\frac{f(y)-f(x)}{y-x} \ \leq \ \frac{f(y)-f(y-h)}{h} \ = \ \frac{f(y-h)-f(y)}{-h},\]

e passando al limite per h\to 0^+ otteniamo:

(56)   \begin{equation*}  	\frac{f(y)-f(x)}{y-x} \leq f^{\prime}(y) \; . 	\end{equation*}

Mettendo insieme (55) e (56) abbiamo la tesi.

\Leftarrow” Per ipotesi f^{\prime} è debolmente crescente, quindi, grazie al punto 2) del lemma 4.6 ci basterà mostrare che \forall \ a<b<c \; con a,b,c \in I, vale r_f(a,b)\leq r_f(b,c).

Il teorema di Lagrange ci dice che esistono \xi_{1} \in (a,b) e \xi_{2} \in (b,c) tali che:

    \[r_f(a,b) \ = \ f^{\prime}(\xi_{1}) \; \text{ e } \; r_f(b,c) = f^{\prime}(\xi_{2}),\]

e dalla debole crescenza di f^{\prime} abbiamo:

    \[r_f(a,b) \ = \ f^{\prime}(\xi_{1}) \ \leq \ f^{\prime}(\xi_{2}) \ = \ r_f(b,c),\]

che è proprio la tesi.

Resta quindi da dimostrare soltanto il teorema 4.5.

Dimostrazione. Per il lemma 4.7 abbiamo che:

    \[f \text{ convessa} \iff f^{\prime} \text{ è debolmente crescente in } I .\]

Inoltre, dal Lemma 3.2 sappiamo che

    \[f^{\prime} \text{ è debolmente crescente in } I \iff (f^{\prime})^{\prime} \geq 0 \text{ in } I  .\]

Allora, dal fatto che (f^{\prime})^{\prime} = f^{\prime \prime}, otteniamo la tesi.

Per il caso della concavità basta ricordare che:

    \[f \text{ è concava} \overset{\rm def}{\iff} -f \text{ è convessa} \iff (-f)^{\prime \prime} \geq 0 \iff -f^{\prime \prime} \geq 0 \iff f^{\prime \prime} \leq 0 .\]


Differenziabilità.

Un altro concetto strettamente legato a quello di derivata è quello di differenziabilità. Per esprimere tale concetto, utilizziamo la nozione di o-piccolo, che viene trattata separatamente, cf. simboli di Landau.

    \[\quad\]

Definizione 4.8. Sia f\colon A  \subseteq \mathbb{R}\to \mathbb{R}. Dato x_{0} \in A un punto di accumulazione per A, si dice che f è differenziabile in x_{0} se esiste un \alpha \in \mathbb{R} tale che:

    \[f(x_{0}+h)  =  f(x_{0})+ \alpha h + o(h), \quad \text{per } h \to 0 .\]

    \[\quad\]

Il legame tra derivabilità e differenziabilità è molto stretto. Il prossimo risultato mostra che queste due nozioni sono equivalenti. Osserviamo da subito che, sebbene tale nozione sia stata inserita in un capitolo a parte e solo dopo numerosi risultati, alcuni autori preferiscono dare sin dal principio la definizione di derivabilità in termini di differenziabilità.

Teorema 4.9. Siano f\colon A  \subseteq \mathbb{R}\to \mathbb{R} e x_{0} \in A un punto di accumulazione per A. Allora:

    \[f \text{ è derivabile in } x_{0} \iff f \text{ è differenziabile in } x_{0}, \mbox{ e si ha } \alpha = f'(x_0).\]

    \[\quad\]

Dimostrazione. f è derivabile in x_{0} se e solo se

    \[f^{\prime}(x_{0}) \ = \ \lim_{h\to0} \frac{f(x_{0}+h)-f(x_{0})}{h} \; \in \mathbb{R} ,\]

ovvero, se e solo se

    \[\lim_{h\to0} \left[ \frac{f(x_{0}+h)-f(x_{0})}{h} - f^{\prime}(x_{0}) \right] \ = \ 0 ,\]

cioè

    \[\lim_{h\to0} \frac{f(x_{0}+h)-f(x_{0})-f^{\prime}(x_{0}) \cdot h}{h} \ = \ 0.\]

In altre parole, f è derivabile in x_0 se e solo se

    \[f(x_{0}+h)-f(x_{0})-f^{\prime}(x_{0}) \cdot h = o(h),\]

ovvero f è differenziabile in x_0 (con \alpha=f'(x_0)).


 
 

Esercizi svolti

In quest’ultima sezione si presenteranno degli esercizi svolti riguardanti gli argomenti trattati nella dispensa.
 
 

Esercizi base sulla derivazione

In questa sezione vedremo come applicare le regole di derivazione introdotte nella sezione precedente. Cominceremo con il calcolo di derivate di funzioni che non presentano particolari difficoltà in modo da rendere chiaro l’uso dei teoremi esposti e dei ragionamenti utili per ottenere il risultato.
 
 

Esercizio 5.1  (\bigstar\largewhitestar\largewhitestar\largewhitestar\largewhitestar). Calcolare la derivata della seguente funzione:

    \[f \colon \mathbb{R} \to \mathbb{R} , \; f(x)=x^3+4x+1.\]

Svolgimento.

Riconosciamo che f è la somma di tre funzioni facilmente riconducibili alle derivate notevoli introdotte nel primo capitolo.

Applicando il teorema 2.1 otteniamo

(57)   \begin{equation*} 	\left[x^3+4x+1\right]'=[x^3]'+[4x]'+[1]'. 	\end{equation*}

Calcoliamo separatamente la derivata delle tre funzioni:

La prima funzione è una potenza, applicando ad esempio (12) con \alpha=3 otteniamo

    \[[x^3]'=3x^{3-1}=3x^2.\]

La seconda funzione è il prodotto di una funzione per una costante, applicando (12) con \alpha=1 abbiamo

    \[[4x]'=4[x]'=4\cdot 1=4.\]

La terza funzione è una costante, applicando (12) con \alpha=0 abbiamo

    \[[1]'=0.\]

Sostituendo in (57) si ha:

    \[\left[x^3+4x+1\right]'=[x^3]'+[4x]'+[1]'=3x^2+4+0=3x^2+4.\]

Osservazione 5.2. Generalizzando il procedimento di questo esercizio si ottiene una formula per il calcolo della derivata di un polinomio:

(58)   \begin{equation*} \left[\sum_{k=0}^n{a_kx^k}\right]^\prime=\sum_{k=1}^n{k\cdot a_kx^{k-1}} . \end{equation*}


 
 

Esercizio 5.3  (\bigstar\largewhitestar\largewhitestar\largewhitestar\largewhitestar). Calcolare la derivata della seguente funzione:

    \[f \colon (0, +\infty) \to \mathbb{R} , \; f(x)=x\cdot \ln(x).\]

Svolgimento.

Questa funzione può essere vista come prodotto di due funzioni g,h \colon (0, +\infty) \to \mathbb{R}, date da

    \[g(x)=x\quad e\quad h(x)=\ln(x) \quad \forall x \in (0,+\infty).\]

Applicando il teorema 2.3 otteniamo:

    \[f'(x)=g'(x)\cdot h(x)+g(x)\cdot h'(x).\]

Per completare il calcolo, occorre determinare g'(x) e h'(x). Notiamo che conosciamo già l’espressione delle derivate di entrambe le funzioni:

    \[\quad\]

  • applicando (12) con \alpha=1 otteniamo g'(x)=\left[x\right]'=1;
  •  

  • applicando (32) otteniamo h'(x)=\left[\ln(x)\right]'=\dfrac{1}{x}.

da cui

    \[\begin{aligned} 	f'(x)&=g'(x)\cdot h(x)+g(x)\cdot h'(x)=\\ 	&=1\cdot \ln(x)+x\cdot \frac{1}{x}=\\ 	&=\ln(x)+\frac{x}{x}=\ln(x)+1. \end{aligned}\]


 
 

Esercizio 5.4  (\bigstar\largewhitestar\largewhitestar\largewhitestar\largewhitestar). Calcolare la derivata della seguente funzione:

    \[f \colon \mathbb{R} \setminus \left\{ \dfrac{\pi}{2} + k \pi\colon k \in \mathbb{Z} \right\} \to \mathbb{R} , \; f(x)=\tan(x).\]

    \[\quad\]

Svolgimento.

Abbiamo già calcolato nell’esempio 2.18 la derivata della tangente applicando la definizione di derivata e sfruttando le formule trigonometriche. Mostriamo ora un modo differente per giungere al risultato che consiste nell’utilizzare la definizione della tangente come il quoziente \tan(x)=\dfrac{\sin(x)}{\cos(x)} e quindi sfruttare la regola di derivazione dei quozienti 2.5. Ponendo g(x)=\sin(x) e h(x)=\cos(x), otteniamo

    \[f'(x)=\left[\frac{g(x)}{h(x)}\right]'=\frac{g'(x)\cdot h(x)-g(x)\cdot h'(x)}{[h(x)]^2}.\]

Applichiamo quindi (15) e (16), ottenendo

    \[\begin{aligned} 	f'(x)&=\frac{g'(x)\cdot h(x)-g(x)\cdot h'(x)}{[h(x)]^2}=\\[0.4em] 	&=\frac{\cos(x)\cdot \cos(x)-\sin(x)\cdot(-\sin(x))}{[\cos(x)]^2}=\\[0.4em] 	&=\frac{\cos^2(x)+\sin^2(x)}{\cos^2(x)} 	=\frac{1}{\cos^2(x)}, 	\end{aligned}\]

che è proprio quello che ci aspettavamo.


 
 

Esercizio 5.5  (\bigstar\bigstar\largewhitestar\largewhitestar\largewhitestar). Calcolare la derivata della seguente funzione:

    \[f \colon \left( - \dfrac{\pi}{2} , \dfrac{\pi}{2}\right) \to \mathbb{R} , \; f(x)=\ln(\cos(x)).\]

Svolgimento.

Osserviamo che f è data dalla composizione delle funzioni h e g, ovvero f=g \circ h, dove

    \[h\colon \left( - \dfrac{\pi}{2} , \dfrac{\pi}{2}\right) \to (0,1], \; h(x)=\cos(x)\]

e

    \[g\colon \left( 0, + \infty \right) \to \mathbb{R}, \; g(x)=\ln(x)\]

Applicando (11) otteniamo che per ogni x \in  \left( - \dfrac{\pi}{2} , \dfrac{\pi}{2}\right) si ha

    \[f'(x)=[g(h(x))]'=g'(h(x))\cdot h'(x).\]

Applichiamo quindi (16) e (32) ottenendo

    \[\begin{aligned} 	f'(x)&=g'(h(x))\cdot h'(x)\\[0.4em] 	&=\dfrac{1}{h(x)}\cdot(-\sin(x))=\\[0.4em] 	&=\dfrac{1}{\cos(x)}\cdot(-\sin(x))=-\tan(x). 	\end{aligned}\]

L’esercizio poteva essere risolto anche applicando direttamente (46): posto f(x)=\ln(h(x)), si ha

    \[f'(x)=\dfrac{h'(x)}{h(x)}=\dfrac{-\sin(x)}{\cos(x)}=-\tan(x).\]


 
 

Esercizio 5.6  (\bigstar\bigstar\largewhitestar\largewhitestar\largewhitestar). Calcolare la derivata della seguente funzione:

    \[f \colon (0, + \infty) \to \mathbb{R} , \; f(x)=\vert \ln(x)\vert.\]

    \[\quad\]

Svolgimento.

Dato che

    \[\ln(x)\geq0\quad \Leftrightarrow\quad x\geq1,\]

otteniamo che

    \[f(x)= 	\begin{cases} 	\ln(x), &  \text{se }x\geq1; \\ 	-\ln(x), &  \text{se }0<x<1. 	\end{cases}\]

Dalla (48) e dalla (32) otteniamo:

    \[f'(x)= 	\begin{cases} 	\left[\ln(x)\right]', &  \text{se } x>1;\\\\ 	\left[-\ln(x)\right]', &  \text{se }0<x<1. 	\end{cases} 	= 	\begin{cases} 	\dfrac{1}{x}, &  \text{se }x>1; \\\\ 	-\dfrac{1}{x}, &  \text{se }0<x<1. 	\end{cases}\]

In alternativa, avremmo potuto calcolare tale derivata senza spezzare il modulo, come osservato in (49), ottenendo

    \[f'(x)=\frac{\ln(x)}{\vert\ln(x)\vert}\cdot\left[\ln(x)\right]'= 	\frac{\ln(x)}{\vert\ln(x)\vert}\cdot\frac{1}{x}.\]

Verifichiamo infine che in x=1 la funzione non è derivabile. Osserviamo che siamo nelle ipotesi del teorema 3.6 e che abbiamo

    \[f'_{-}(1)= \lim_{ x \to 1^-} f'(x)= -1 \quad \mbox{e} \quad 	f'_{+}(1)=\lim_{ x \to 1^+} f'(x)= 1.\]


 
 

Esercizi misti

Nella sezione precedente abbiamo familiarizzato con i teoremi di derivazione e con alcune delle derivate notevoli, adesso vediamo come applicare i metodi visti finora ad esercizi più complicati.

    \[\quad\]

Esercizio 5.7  (\bigstar\bigstar\largewhitestar\largewhitestar\largewhitestar). Determinare l’equazione della retta tangente al grafico della funzione f nel punto (x_0, f(x_0)), nei seguenti casi:

    \[\quad\]

  1. f(x)=-\dfrac{1}{3x}, \qquad x_0=1;
  2.  

  3. f(x)=\ln(2x-1), \qquad x_0=1;
  4.  

  5. f(x)=-\arctan(x+1), \qquad x_0=0;
  6.  

  7. f(x)=x^2+ \sqrt{1+e^{-x}}, \qquad x_0=0.

    \[\quad\]

Svolgimento punto 1.

Prima di tutto ricaviamo l’ordinata y_0 corrispondente all’ascissa x_0=1:

    \[y_0 =f(x_0)= - \dfrac{1}{3x_0} = -\dfrac{1}{3}.\]

Applicando 2.1 e (12) con \alpha=-1 otteniamo

    \[f^\prime(x) = \dfrac{1}{3x^2}.\]

Quindi,

    \[f^\prime(x_0) = \dfrac{1}{3x_0^2} = \dfrac{1}{3}.\]

Allora la retta tangente, cf. (3), è


Svolgimento punto 2.

L’ordinata y_0 corrispondente all’ascissa x_0=1 è

    \[y_0 = f(x_0)= \ln(2x_0-1) = \ln(1)=0.\]

Applicando 2.4 e (32) otteniamo

    \[f^\prime(x) = \dfrac{2}{2x-1}.\]

Quindi,

    \[f^\prime(x_0) =  \dfrac{2}{2x_0-1}= 2.\]

Allora la retta tangente, cf. (3), è


Svolgimento punto 3.

L’ordinata y_0 corrispondente all’ascissa x_0=0 è

    \[y_0 = f(x_0)=-\arctan(x_0+1) = -\arctan(1)=-\dfrac{\pi}{4}.\]

Applicando 2.4 e (21) otteniamo

    \[f^\prime(x) = -\dfrac{1}{(x+1)^2+1}.\]

Quindi,

    \[f^\prime(x_0) = -\dfrac{1}{(x_0+1)^2+1}= -\frac 1 2.\]

Allora la retta tangente, cf. (3), è

    \[\boxcolorato{analisi}{y =-\dfrac{\pi}{4}-\frac 1 2x}\]


Svolgimento punto 4.

L’ordinata y_0 corrispondente all’ascissa x_0=0 è

    \[y_0 = f(x_0)=x_0^2+ \sqrt{1+e^{-x_0}} = \sqrt{2}.\]

Applicando le regole 2.1 e 2.4 e le derivate notevoli (12) con \alpha=2, (29) e (31) otteniamo

    \[f^\prime(x) = 2x-\dfrac{e^{-x}}{2\sqrt{1+e^{-x}}}.\]

Quindi,

    \[f^\prime(x_0) = 2x_0-\dfrac{e^{-x_0}}{2\sqrt{1+e^{-x_0}}}= - \dfrac{1}{2\sqrt{2}} .\]

Allora la retta tangente, cf. (3), è

    \[\boxcolorato{analisi}{y =\sqrt{2}- \dfrac{1}{2\sqrt{2}}x}\]


 
 

Esercizio 5.8  (\bigstar\bigstar\largewhitestar\largewhitestar\largewhitestar). Determinare a e b in modo che la curva di equazione y=a \sin 2x + b \cos x + 1 intersechi l’asse x nel punto di coordinate (\pi,0), avendo in tale punto tangente che forma con l’asse x un angolo di 135^\circ.

    \[\quad\]

Svolgimento.

Le due condizioni imposte alla curva si traducono in due equazioni. La prima equazione si ricava imponendo il passaggio della curva per il punto (\pi,0):

(59)   \begin{equation*} 		0 = a \sin 2\pi + b \cos \pi + 1 \quad \iff \quad 0 = - b  + 1 \quad \iff \quad b=1. 	\end{equation*}

Per imporre la seconda condizione, calcoliamo il coefficiente angolare della retta tangente alla curva in (\pi,0). Poiché la curva data è il grafico della funzione

    \[f\colon \mathbb{R} \to \mathbb{R}, \; f(x)=a \sin 2x + b \cos x + 1,\]

il coefficiente angolare della retta tangente alla curva nel punto dato corrisponde alla derivata della funzione f nel punto x_0=\pi:

(60)   \begin{equation*} 		f^\prime(x_0) = 2a \cos 2x_0 - b \sin x_0 \quad \Longrightarrow \quad f^\prime(\pi) = 2a. 	\end{equation*}

Inoltre, siccome il coefficiente angolare di una retta è la tangente dell’angolo che essa forma con il semiasse positivo delle x, la seconda condizione è equivalente a

(61)   \begin{equation*} 		\tan(135^\circ) = 2a \quad \iff \quad -1 = 2a \quad \iff \quad a=-\dfrac{1}{2}. 	\end{equation*}

Quindi i valori cercati sono

    \[\boxcolorato{analisi}{a=-\dfrac{1}{2}, \qquad b=1.}\]


 
 

Esercizio 5.9  (\bigstar\bigstar\largewhitestar\largewhitestar\largewhitestar). la derivata della seguente funzione:

    \[f \colon \mathbb{R} \setminus \{ -1 \} \to \mathbb{R} , \; f(x)=\frac{x\sin(x)}{x+1}.\]

    \[\quad\]

Svolgimento.

(62)   \begin{equation*} 		f(x)=\frac{x \sin x + \sin x - \sin x}{x+1} 		= 		\frac{(x+1)\sin x - \sin x}{x+1} 		= 		\sin x - \frac{\sin x}{x+1} 		\qquad 		\forall x \neq -1. 	\end{equation*}

Ponendo g,h \colon\mathbb{R} \to \mathbb{R} le funzioni definite da

(63)   \begin{equation*} 		g(x)=\sin x, 		\quad 		h(x) = x+1 		\qquad 		\forall x \in \mathbb{R}, 	\end{equation*}

si ha quindi

(64)   \begin{equation*} 		f(x) 		= 		g(x) + \frac{g(x)}{h(x)} 		\qquad 		\forall x \neq -1. 	\end{equation*}

Osservando che per ogni x \in \mathbb{R} si ha g'(x)= \cos x e h'(x)=1, utilizzando il teorema 2.1 e la proposizione 2.5 si ottiene

(65)   \begin{equation*} 		\begin{split} 			f'(x) 			= & 			g'(x) 			- \frac{g'(x)h(x)-g(x)h'(x)}{h^2(x)}= 			\\ 			= & 			\cos x 			- 			\frac{(x+1) \cos x - \sin x}{(x+1)^2}= 			\\ 			=& 			\frac{\sin x + x(x+1)\cos x}{(x+1)^2} 			\qquad 			\forall x \neq -1. 		\end{split} 	\end{equation*}


 
 

Esercizio 5.10  (\bigstar\bigstar\largewhitestar\largewhitestar\largewhitestar). la derivata della seguente funzione:

    \[f \colon [-4,+\infty) \to \mathbb{R} , \; f(x)=\sqrt{\ln(x+5)}.\]

    \[\quad\]

Svolgimento.

Risulta f=k \circ h \circ g, dove

    \begin{gather*} 	g \colon [-4,+\infty) \to [1,+\infty), \; g(x)=x+5, \\ h \colon [1,+\infty) \to [0,+\infty), \;	h(y)= \ln y, \\ k \colon [0,+\infty) \to [0,+\infty), \;	k(z)= \sqrt{z}. \end{gather*}

In altre parole, si ha f(x)=k(h(g(x))) per ogni x \geq -4. Applicando due volte il teorema 2.4, prima alla funzione k \circ (h \circ g) e poi alla funzione h \circ g, si ottiene

(66)   \begin{equation*} 	f'(x) 	= 	k'\big((h\circ g)(x) \big) (h\circ g)'(x) 	= 	k'(h(g(x))) 	h'(g(x))g'(x) 	\qquad 	\forall x \in [-4,+\infty). \end{equation*}

Osservando che g'(x)=1, h'(y)=\dfrac{1}{y} e k'(z)=\dfrac{1}{2\sqrt{z}} e sostituendo in (66), si ottiene

(67)   \begin{equation*} 	f'(x) 	= 	\frac{1}{2\sqrt{\ln(x+5)}}\frac{1}{x+5}. \end{equation*}


 
 

Esercizio 5.11  (\bigstar\bigstar\largewhitestar\largewhitestar\largewhitestar). Calcolare la derivata della seguente funzione:

    \[f \colon (0, \pi) \to \mathbb{R} , \; f(x)=(\sin(x))^{\cos(x)}.\]

    \[\quad\]

Svolgimento.

Quando si hanno delle funzioni all’esponente la scelta più conveniente è osservare che

    \[\boxed{y=f(x) \quad\text{con}\,\,f(x)>0\iff y=e^{\ln(f(x))}. }\]

In questo modo otteniamo due vantaggi immediati:

    \[\quad\]

  • possiamo sfruttare le proprietà del logaritmo per trasformare l’esponente dell’argomento in coefficiente

        \[\ln(a^b)=b\cdot \ln(a)\quad \text{con}\,\,a>0\]

  •  

  • otteniamo una composizione di funzioni semplici da trattare.

In questo esercizio applichiamo la trasformazione e otteniamo

    \[f(x)=\left(\sin\left(x\right)\right)^{\cos\left(x\right)}=e^{\ln\left(\sin\left(x\right)^{\cos\left(x\right)}\right)}=e^{\cos(x)\cdot \ln\left(\sin\left(x\right)\right)}.\]

Notiamo che ci sono essenzialmente quattro funzioni di cui tenere conto:

    \begin{gather*} 	g\colon \mathbb{R} \to \mathbb{R}, \; g(x)=\sin(x); \quad h\colon (0, +\infty) \to \mathbb{R}, \; h(y)=\ln(y); \\ 	k\colon \mathbb{R} \to \mathbb{R}, \; k(z)=\cos(z); \quad p\colon \mathbb{R} \to \mathbb{R}, \;  p(w)=e^w. \end{gather*}

Infatti, si ha

(68)   \begin{equation*} 		f=p \circ \big( k \cdot (h\circ g) \big), 	\end{equation*}

dunque applicando il teorema 2.3 e il teorema 2.4 a (68), si ottiene

(69)   \begin{equation*} 		\begin{split} 			f'(x) 			= & 			p'\big( k(x) \cdot (h\circ g)(x) \big) \cdot \big( k \cdot (h\circ g) \big)'(x)= 			\\ 			= & 			p'\big( k(x) \cdot h(g((x)) \big) \cdot \big( k'(x) (h\circ g)(x) + k(x)(h\circ g)'(x) \big)= 			\\ 			= & 			p'\big( k(x) \cdot h(g((x)) \big) \cdot \big( k'(x) h(g((x)) + k(x) h'g((x))g'(x) \big), 		\end{split} 	\end{equation*}

dove nella prima uguaglianza si è applicato il teorema 2.4 alla funzione p \circ \big( k \cdot (h\circ g) \big), nella seconda uguaglianza si è applicato il teorema 2.3 alla funzione k \cdot (h \circ g), e infine nella terza uguaglianza si è applicato il teorema 2.4 alla funzione h \circ g. Osservando che

(70)   \begin{equation*} 		p'(z)=e^z,\quad 		k'(x)= -\sin x, \quad 		h'(y)= \frac{1}{y}, \quad 		g'(x) = \cos x 	\end{equation*}

nei rispettivi domini e sostituendo in (69), si ottiene

(71)   \begin{equation*} 		f'(x) 		= 		e^{\cos x \ln(\sin x)} 		\big( 		-\sin x \ln(\sin x) + \frac{\cos^2 x}{\sin x}  		\big) 		\qquad 		\forall x \in (0,\pi). 	\end{equation*}


 
 

Esercizio 5.12  (\bigstar\bigstar\largewhitestar\largewhitestar\largewhitestar). Calcolare la derivata della seguente funzione:

    \[f \colon \mathbb{R} \setminus \left\{ \frac 5 3 \right \}\to \mathbb{R} , \; f(x)=\sqrt[5]{\frac{\sin(x^2)}{5-3x}}.\]

    \[\quad\]

Svolgimento.

Si vede che f è una composizione di funzioni:

(72)   \begin{equation*} 	f= k \circ \left(\frac{g \circ h}{p} \right), \end{equation*}

dove k,g,h,p \colon \mathbb{R} \to \mathbb{R} sono definite da

(73)   \begin{equation*} 	k(z)=\sqrt[5]{z}, 	\quad 	g(y)= \sin y, 	\quad 	h(x)=x^2, 	\quad 	p(x)= 5 - 3x 	\qquad 	\forall x,y,z \in \mathbb{R}. \end{equation*}

Si ha quindi

(74)   \begin{equation*} 	\begin{split} 		f' 		= & 		k' \circ \left(\frac{g \circ h}{p} \right) 		\cdot 		\left(\frac{g \circ h}{p} \right)'= 		\\ 		= & 		k' \circ \left(\frac{g \circ h}{p} \right) 		\cdot 		\frac{(g \circ h)'p - (g\circ h)p'}{p^2}= 		\\ 		=& 		k' \circ \left(\frac{g \circ h}{p} \right) 		\cdot 		\frac{(g' \circ h)h'p - (g\circ h)p'}{p^2}, 	\end{split} \end{equation*}

dove nella prima uguaglianza si è applicato il teorema 2.4 alla composizione k \circ \left(\dfrac{g \circ h}{p} \right), nella seconda uguaglianza si è applicato la proposizione 2.6 al quoziente \dfrac{g \circ h}{p}, mentre l’ultima uguaglianza segue dal teorema 2.4 applicato alla composizione g \circ h. Osservando che, rispettivamente per (12) con \alpha=\dfrac 1 5, (15) e (12) con \alpha=2, si ha

(75)   \begin{equation*} 	k'(z)= \frac{1}{5 \sqrt[5]{z^4}}, 	\quad 	g'(y)=\cos y, 	\quad 	h'(x)=2x, 	\quad 	p'(x)=-3 	\qquad 	\forall x,y \in \mathbb{R}, \,\, \forall z \neq 0, \end{equation*}

sostituendo in (76) si ottiene

(76)   \begin{equation*} 	f'(x) 	= 	\frac{1}{5 \sqrt[5]{\left( \dfrac{\sin x^2}{5-3x}\right)^4}} 	\cdot 	\frac{\cos x^2 (5-3x) +3 \sin x^2}{(5-3x)^2} 	\quad 	\forall x \in \mathbb{R} \setminus \left( \left\{\frac{5}{3}\right\} \cup \{x \in \mathbb{R} \colon x=\pm\sqrt{n\pi},\,n \in \mathbb{N} \} \right). \end{equation*}

Per quanto riguarda i punti in \{x \in \mathbb{R} \colon x=\pm\sqrt{n\pi},\,n \in \mathbb{N} \}, da (76) segue che per ogni n \in \mathbb{N} il limite di f'(x) per x \to \pm \sqrt{n\pi} è infinito. Dunque, per il teorema 3.6, f non è derivabile in \pm \sqrt{n\pi} per ogni n \in \mathbb{N}.


 
 

Esercizio 5.13  (\bigstar\bigstar\largewhitestar\largewhitestar\largewhitestar). Calcolare la derivata della funzione

    \[f\colon \mathbb{R} \to \mathbb{R},\;  f(x)=2\arctan(x)-\arcsin\left(\dfrac{2x}{1+x^2}\right).\]

    \[\quad\]

Svolgimento.

Osserviamo innanzitutto che la funzione f è ben definita in quanto la funzione

    \[k\colon \mathbb{R} \to \mathbb{R}, \; k(x)= \frac{2x}{1+x^2}\]

assume valori in [-1,1], e quindi è ben definita la composizione g \circ k, dove g\colon [-1,1] \to \mathbb{R},\; g(y)= \arcsin(y). Infatti, applicando la disuguaglianza

    \[\left( |x| - |y| \right)^2 \geq 0 \iff 2|xy| \leq x^2+y^2 \qquad \forall x,y \in \mathbb{R},\]

con y=1, otteniamo che

    \[\left\vert  \frac{2x}{1+x^2}\right\vert \leq 1.\]

Applicando il teorema 2.1 otteniamo

    \[f'(x)=2\left[\arctan(x)\right]'-\left[\arcsin\left(\dfrac{2x}{1+x^2}\right)\right]'.\]

Per la prima derivata, applichiamo (21) e otteniamo

    \[2\left[\arctan(x)\right]'=\dfrac{2}{1+x^2}.\]

La seconda è la composizione delle funzioni g,k definite sopra. Utilizzando il teorema 2.4 otteniamo

    \[\left[g(k(x))\right]'=g'(k(x))\cdot k'(x).\]

Da (19) si ha che

    \[g'(y)=\frac{1}{\sqrt{1-y^2}},\]

e sostituendo y=k(x):

    \[\begin{aligned} 	g'(k(x))&=\dfrac{1}{\sqrt{1-k^2(x)}}=\dfrac{1}{\sqrt{1-\left(\dfrac{2x}{1+x^2}\right)^2}}= 	\dfrac{1}{\sqrt{1-\dfrac{4x^2}{\left(1+x^2\right)^2}}}=\\ 	&=\dfrac{1}{\sqrt{\dfrac{\left(1+x^2\right)^2-4x^2}{\left(1+x^2\right)^2}}}= 	\dfrac{1+x^2}{\sqrt{\left(1+x^2\right)^2-4x^2}}= 	\dfrac{1+x^2}{\sqrt{1+2x^2+x^4-4x^2}}=\\ 	&=\dfrac{1+x^2}{\sqrt{1-2x^2+x^4}}=\dfrac{1+x^2}{\sqrt{\left(1-x^2\right)^2}}=\dfrac{1+x^2}{\vert{1-x^2}\vert}. 	\end{aligned}\]

Per calcolare la derivata di k, usiamo la proposizione 2.6, ottenendo:

    \[\begin{aligned} 	\left[\frac{2x}{1+x^2}\right]'&=\dfrac{\left[2x\right]'\cdot\left(1+x^2\right)-2x\cdot\left[1+x^2\right]'}{\left(1+x^2\right)^2}= 	\\ 	&= 	\dfrac{2\left(1+x^2\right)-2x\cdot\left(2x\right)}{\left(1+x^2\right)^2}=\dfrac{2-2x^2}{\left(1+x^2\right)^2}=\dfrac{2\left(1-x^2\right)}{\left(1+x^2\right)^2}. 	\end{aligned}\]

Sostituendo nell’equazione iniziale si ottiene

    \[\begin{aligned} 	\left[g(k(x))\right]'=g'(k(x))\cdot k'(x)=\dfrac{1+x^2}{\vert{1-x^2}\vert}\cdot\dfrac{2\left(1-x^2\right)}{\left(1+x^2\right)^2}= 	\dfrac{2\left(1-x^2\right)}{\vert{1-x^2}\vert\cdot\left(1+x^2\right)}. 	\end{aligned}\]

Otteniamo infine la derivata di f(x):

    \[\begin{aligned} 	f'(x)&=\left[2\arctan(x)\right]'-\left[\arcsin\left(\frac{2x}{1+x^2}\right)\right]'=\\ 	&=\dfrac{2}{1+x^2}-\dfrac{2\left(1-x^2\right)}{\vert{1-x^2}\vert\cdot\left(1+x^2\right)}. 	\end{aligned}\]


 
 

Esercizio 5.14  (\bigstar\bigstar\largewhitestar\largewhitestar\largewhitestar). Siano

    \[B=\left\{ x \in \mathbb{R}\colon x= \pm \sqrt{n \pi},\, n \in \mathbb{N}\right\} \cup  					\left\{ x \in \mathbb{R}\colon x= \pm \sqrt{ \dfrac{\pi}{2}+ n\pi},\, n \in \mathbb{N}\right\} \qquad \text{e} \qquad A= \mathbb{R} \setminus B.\]

Calcolare la derivata della funzione

    \[f\colon A \to \mathbb{R}, \; f(x)=\ln^2\left(\tan^2\left(x^2\right)\right).\]

Svolgimento.

Si ha f= g \circ h \circ k, dove

    \begin{gather*} 		g\colon (0, +\infty) \to \mathbb{R}, \; g(z)=\ln^2(z); \qquad h\colon \mathbb{R} \setminus \left\{ \dfrac{\pi}{2}+ k \pi, \, k \in \mathbb{Z}\right\} \to \mathbb{R}, \; h(y)=\tan^2(y);\\ 		k\colon A \to \mathbb{R}, \; k(x)=x^2, 		\end{gather*}

e poiché g,h,k sono derivabili nei rispettivi domini, si ha

(77)   \begin{equation*} 		f' 		= 		g' \circ (h \circ k) \cdot (h \circ g)' 		= 		g' \circ (h \circ k) \cdot( h' \circ g) \cdot g', 	\end{equation*}

dove nella prima uguaglianza abbiamo usato il teorema 2.4 per la funzione g \circ (h \circ k) e nella seconda uguaglianza abbiamo applicato il teorema 2.4 alla funzione h \circ k. Per calcolare g',h', applichiamo il teorema 2.4, (12) con \alpha=2, (32) e (16):

(78)   \begin{equation*} 		g'(z)= 		2\frac{\ln z}{z}, 		\qquad 		h'(y) 		=2 \frac{\tan y}{\cos^2 y} 	\end{equation*}

nei rispettivi domini. Sostituendo (78) in (77), si ottiene

(79)   \begin{equation*} 		g'(x) 		= 		2\frac{\ln (\tan^2 x^2)}{ \tan^2 x^2} 		\cdot 		2 \frac{\tan x^2}{\cos^2 x^2} 		\cdot 		2 x 		= 		8 \frac{x \ln (\tan^2 x^2)}{\sin x \cos x} 		\qquad 		\forall x \in A. 	\end{equation*}


 
 

Esercizio 5.15  (\bigstar\bigstar\largewhitestar\largewhitestar\largewhitestar). Calcolare la derivata della funzione

    \[f\colon (-\infty, 0) \to \mathbb{R}, \; f(x)=\ln\left(\dfrac{1-e^x}{e^x}\right)+\dfrac{1}{e^x-1}.\]

Svolgimento.

Notiamo che f è la somma di due funzioni, quindi applichiamo il teorema 2.1 ottenendo

    \[f'(x)= 	\left[\ln\left(\dfrac{1-e^x}{e^x}\right)\right]'+\left[\dfrac{1}{e^x-1}\right]'.\]

Calcoliamo la derivata di \dfrac{1}{e^x-1} utilizzando il teorema 2.4, (12) con \alpha=-1 e (31):

    \[\left[\dfrac{1}{e^x-1}\right]'= - (e^x-1)^{-2}[e^x-1]'= -\frac{e^x}{(e^x-1)^2}.\]

Deriviamo adesso la funzione \ln\left(\dfrac{1-e^x}{e^x}\right); applicando le proprietà dei logaritmi otteniamo

    \[\ln\left(\dfrac{1-e^x}{e^x}\right)= \ln\left(1-e^x\right)- x.\]

Applicando il teorema 2.1 e (11) e le derivate notevoli (12) con \alpha=1, (32) e (31), otteniamo:

    \[\left[  \ln\left(\dfrac{1-e^x}{e^x}\right) \right]'= [ \ln\left(1-e^x\right)]'- [x]'= -\frac{e^x}{1-e^x} - 1= \frac{1}{e^x-1}.\]

Concludiamo perciò che

    \[\begin{aligned} 	f'(x)&= 	\left[\ln\left(\dfrac{1-e^x}{e^x}\right)\right]'+\left[\dfrac{1}{e^x-1}\right]'=\\ 	&=\dfrac{1}{e^x-1}-\dfrac{e^x}{\left(e^x-1\right)^2}=-\dfrac{1}{\left(e^x-1\right)^2}.	 	\end{aligned}\]


 
 

Esercizio 5.16  (\bigstar\bigstar\largewhitestar\largewhitestar\largewhitestar). Calcolare la derivata della funzione

    \[f\colon \Big(-\infty, \dfrac{\ln 2}{2}\Big] \to \mathbb{R}, \; f(x)=\arccos\vert e^{2x}-1\vert.\]

Svolgimento.

Possiamo procedere utilizzando due metodi equivalenti: il primo consiste nello sviluppare il modulo e poi derivare la funzione, il secondo nell’utilizzo della formula (49) e quindi nel lasciare il modulo in forma compatta.

Scegliamo di utilizzare il secondo metodo. Innanzitutto, notiamo che f è ben definita in quanto

    \[\vert e^{2x}-1\vert \leq 1 \iff x \leq  \dfrac{\ln 2}{2}.\]

Osserviamo inoltre che f=g\circ h, dove

    \[g\colon [-1,1] \to \mathbb{R}, \; g(y)=\arccos(y);\qquad h\colon \Big(-\infty, \dfrac{\ln 2}{2}\Big] \to [0,1], \; h(x)=|e^{2x}-1|.\]

Applicando il teorema 2.4 e la derivata notevole (20), otteniamo

    \[f'(x)=\left[g\left(h(x)\right)\right]'=g'\left(h(x)\right)\cdot h'(x)=	-\dfrac{h'(x)}{\sqrt{1-\left(e^{2x}-1\right)^2}}.\]

Dalla formula (49) e dalla derivata notevole (31) si ha inoltre

    \[h'(y)=	\dfrac{e^{2x}-1}{\vert e^{2x}-1\vert}\cdot\left[e^{2x}-1\right]' 	=\dfrac{e^{2x}-1}{\vert e^{2x}-1\vert}\cdot2e^{2x}.\]

e dunque

    \[f'(x)= -\dfrac{2e^{2x}}{\sqrt{1-\left(e^{2x}-1\right)^2}}\cdot\dfrac{e^{2x}-1}{\vert e^{2x}-1\vert}.\]

Osserviamo infine che

    \[e^{2x}-1 >0 \Leftrightarrow x >0,\]

dunque possiamo scrivere la derivata in forma estesa come

    \[\begin{aligned} 	f'(x)= 	\begin{cases} 	-\dfrac{2e^{2x}}{\sqrt{1-\left(e^{2x}-1\right)^2}}, & \mbox{ se }x \in \Big(0, \frac{\ln 2}{2}\Big];\\\\ 	\dfrac{2e^{2x}}{\sqrt{1-\left(e^{2x}-1\right)^2}}, & \mbox{ se } x \in(-\infty, 0). 	\end{cases}\\ 	\end{aligned}\]

Per concludere, analizziamo la derivabilità di f nel punto x=0:

    \[\begin{aligned} 	\lim_{x\to0^+}f'(x)&= 	\lim_{x\to0^+}-\dfrac{2e^{2x}}{\sqrt{1-\left(e^{2x}-1\right)^2}}= 	-\dfrac{2e^{2\cdot0}}{\sqrt{1-\left(e^{2\cdot0}-1\right)^2}}=-2,\\\\ 	\lim_{x\to0^-}f'(x)&= 	\lim_{x\to0^-}\dfrac{2e^{2x}}{\sqrt{1-\left(e^{2x}-1\right)^2}}= 	\dfrac{2e^{2\cdot0}}{\sqrt{1-\left(e^{2\cdot0}-1\right)^2}}=2. 	\end{aligned}\]

I due limiti sono diversi, quindi x=0 è un punto di non derivabilità (punto angoloso) per il teorema 3.6.


 
 

Riferimenti bibliografici


[1] Berretti, A., Analisi Matematica 1, UniversItalia, 2018.


[2] Bertsch, M., Dal Passo, R., Giacomelli, R.; Analisi matematica, 2ed, McGraw-Hill, 2011.


[3] Giusti, E., Analisi matematica 1, Programma di matematica fisica elettronica, Bollati Boringhieri 1992.


[4] Rudin, W., Principles of mathematical analysis, International series in pure and applied mathematics, McGraw-Hill 1976.
 
 

Tutta la teoria di analisi matematica

Leggi...

  1. Teoria Insiemi
  2. Il metodo della diagonale di Cantor
  3. Logica elementare
  4. Densità dei numeri razionali nei numeri reali
  5. Insiemi Numerici \left(\mathbb{N},\, \mathbb{Z},\, \mathbb{Q}\right)
  6. Il principio di induzione
  7. Gli assiomi di Peano
  8. L’insieme dei numeri reali: costruzione e applicazioni
  9. Concetti Fondamentali della Retta Reale: Sintesi Teorica
  10. Costruzioni alternative di \mathbb{R}
  11. Binomio di Newton
  12. Spazi metrici, un’introduzione
  13. Disuguaglianza di Bernoulli
  14. Disuguaglianza triangolare
  15. Teoria sulle funzioni
  16. Funzioni elementari: algebriche, esponenziali e logaritmiche
  17. Funzioni elementari: trigonometriche e iperboliche
  18. Funzioni goniometriche: la guida essenziale
  19. Teorema di Bolzano-Weierstrass per le successioni
  20. Criterio del rapporto per le successioni
  21. Definizione e proprietà del numero di Nepero
  22. Limite di una successione monotona
  23. Successioni di Cauchy
  24. Il teorema ponte
  25. Teoria sui limiti
  26. Simboli di Landau
  27. Funzioni continue – Teoria
  28. Il teorema di Weierstrass
  29. Il teorema dei valori intermedi
  30. Il teorema della permanenza del segno
  31. Il teorema di Heine-Cantor
  32. Il teorema di esistenza degli zeri
  33. Il metodo di bisezione
  34. Teorema ponte versione per le funzioni continue
  35. Discontinuità di funzioni monotone
  36. Continuità della funzione inversa
  37. Teorema delle contrazioni o Teorema di punto fisso di Banach-Caccioppoli
  38. Teoria sulle derivate
  39. Calcolo delle derivate: la guida pratica
  40. Teoria sulle funzioni convesse
  41. Il teorema di Darboux
  42. I teoremi di de l’Hôpital
  43. Teorema di Fermat
  44. Teoremi di Rolle e Lagrange
  45. Il teorema di Cauchy
  46. Espansione di Taylor: teoria, esempi e applicazioni pratiche
  47. Polinomi di Taylor nei limiti: istruzioni per l’uso
  48. Integrali definiti e indefiniti
  49. Teorema fondamentale del calcolo integrale (approfondimento)
  50. Integrali ricorsivi
  51. Formule del trapezio, rettangolo e Cavalieri-Simpson
  52. Teoria sugli integrali impropri
  53. Funzioni integrali – Teoria
  54. Introduzione ai numeri complessi – Volume 1 (per un corso di ingegneria — versione semplificata)
  55. Introduzione ai numeri complessi – Volume 1 (per un corso di matematica o fisica)
  56. Serie numeriche: la guida completa
  57. Successioni di funzioni – Teoria
  58. Teoremi sulle successioni di funzioni
    1. 58a. Criterio di Cauchy per la convergenza uniforme
    2. 58b. Limite uniforme di funzioni continue
    3. 58c. Passaggio al limite sotto il segno di integrale
    4. 58d. Limite uniforme di funzioni derivabili
    5. 58e. Piccolo teorema del Dini
    6. 58f. Procedura diagonale e teorema di Ascoli-Arzela
  59. Serie di funzioni – Teoria
  60. Serie di potenze – Teoria
  61. Serie di Fourier – Teoria e applicazioni
  62. Integrali multipli — Parte 1 (teoria)
  63. Integrali multipli — Parte 2 (teoria e esercizi misti)
  64. Regola della Catena — Teoria ed esempi.
  65. Jacobiano associato al cambiamento di coordinate sferiche
  66. Guida ai Massimi e Minimi: Tecniche e Teoria nelle Funzioni Multivariabili
  67. Operatore di Laplace o Laplaciano
  68. Teoria equazioni differenziali
  69. Equazione di Eulero
  70. Teoria ed esercizi sulla funzione Gamma di Eulero
  71. Teoria ed esercizi sulla funzione Beta
  72. Approfondimento numeri complessi
  73. Diverse formulazioni dell’assioma di completezza
  74. Numeri di Delannoy centrali
  75. Esercizi avanzati analisi

 
 

Tutte le cartelle di Analisi Matematica

Leggi...

  1. Prerequisiti di Analisi
    1. Ripasso algebra biennio liceo
    2. Ripasso geometria analitica
    3. Ripasso goniometria e trigonometria
    4. Errori tipici da evitare
    5. Insiemi numerici N,Z,Q,R
    6. Funzioni elementari
    7. Logica elementare
    8. Insiemi
  2. Successioni
    1. Teoria sulle Successioni
    2. Estremo superiore e inferiore
    3. Limiti base
    4. Forme indeterminate
    5. Limiti notevoli
    6. Esercizi misti Successioni
    7. Successioni per ricorrenza
  3. Funzioni
    1. Teoria sulle funzioni
    2. Verifica del limite in funzioni
    3. Limite base in funzioni
    4. Forme indeterminate in funzioni
    5. Limiti notevoli in funzioni
    6. Calcolo asintoti
    7. Studio di funzione senza derivate
    8. Dominio di una funzione
    9. Esercizi misti Funzioni
    10. Esercizi misti sui Limiti
  4. Funzioni continue-lipschitziane-holderiane
    1. Teoria sulle Funzioni continue-lipschitziane-holderiane
    2. Continuità delle funzioni
    3. Continuità uniforme
    4. Teorema degli zeri
    5. Esercizi sul teorema di Weierstrass senza l’uso delle derivate
  5. Calcolo differenziale
    1. Derivate
    2. Calcolo delle derivate
    3. Retta tangente nel calcolo differenziale
    4. Punti di non derivabilità nel calcolo differenziale
    5. Esercizi sul teorema di Weierstrass con l’uso delle derivate
    6. Studio di funzione completo nel calcolo differenziale
    7. Esercizi teorici nel calcolo differenziale
    8. Metodo di bisezione
    9. Metodo di Newton
  6. Teoremi del calcolo differenziale
    1. Teoria sui Teoremi del calcolo differenziale
    2. Teorema di Rolle
    3. Teorema di Lagrange
    4. Teorema di Cauchy
    5. Teorema di De L’Hôpital
  7. Calcolo integrale
    1. Integrale di Riemann
    2. Integrali immediati
    3. Integrale di funzione composta
    4. Integrali per sostituzione
    5. Integrali per parti
    6. Integrali di funzione razionale
    7. Calcolo delle aree
    8. Metodo dei rettangoli e dei trapezi
    9. Esercizi Misti Integrali Indefiniti
    10. Esercizi Misti Integrali Definiti
  8. Integrali impropri
    1. Teoria Integrali impropri
    2. Carattere di un integrale improprio
    3. Calcolo di un integrale improprio
  9. Espansione di Taylor
    1. Teoria Espansione di Taylor
    2. Limiti di funzione con Taylor
    3. Limiti di successione con Taylor
    4. Stime del resto
  10. Funzioni integrali (Approfondimento)
    1. Teoria Funzioni integrali (Approfondimento)
    2. Studio di funzione integrale
    3. Limiti con Taylor e De L’Hôpital
    4. Derivazione di integrali parametrici (Tecnica di Feynmann)
  11. Numeri Complessi
    1. Teoria Numeri complessi
    2. Espressioni con i numeri complessi
    3. Radice di un numero complesso
    4. Equazioni con i numeri complessi
    5. Disequazioni con i numeri complessi
    6. Esercizi misti Numeri complessi
  12. Serie numeriche
    1. Teoria Serie numeriche
    2. Esercizi Serie a termini positivi
    3. Esercizi Serie a termini di segno variabile
    4. Esercizi Serie geometriche e telescopiche
  13. Successioni di funzioni
    1. Teoria Successioni di funzioni
    2. Esercizi Successioni di funzioni
  14. Serie di funzioni
    1. Teoria Serie di funzioni
    2. Esercizi Serie di funzioni
  15. Serie di potenze
    1. Teoria Serie di potenze
    2. Esercizi Serie di potenze
  16. Serie di Fourier
    1. Teoria Serie di Fourier
    2. Esercizi Serie di Fourier
  17. Trasformata di Fourier
    1. Teoria Trasformata di Fourier
    2. Esercizi Trasformata di Fourier
  18. Funzioni di più variabili
    1. Teoria Funzioni di più variabili
    2. Massimi e minimi liberi e vincolati
    3. Limiti in due variabili
    4. Integrali doppi
    5. Integrali tripli
    6. Integrali di linea di prima specie
    7. Integrali di linea di seconda specie
    8. Forme differenziali e campi vettoriali
    9. Teorema di Gauss-Green
    10. Integrali di superficie
    11. Flusso di un campo vettoriale
    12. Teorema di Stokes
    13. Teorema della divergenza
    14. Campi solenoidali
    15. Teorema del Dini
  19. Equazioni differenziali lineari e non lineari
    1. Teoria equazioni differenziali lineari e non lineari
    2. Equazioni differenziali lineari e non lineari del primo ordine omogenee
  20. Equazioni differenziali lineari
    1. Del primo ordine non omogenee
    2. Di ordine superiore al primo,a coefficienti costanti,omogenee
    3. Di ordine superiore al primo,a coefficienti costanti,non omogenee
    4. Di Eulero,di Bernoulli,di Clairaut,di Lagrange e di Abel
    5. Non omogenee avente per omogenea associata un’equazione di Eulero
    6. Sistemi di EDO
  21. Equazioni differenziali non lineari
    1. A variabili separabiliO
    2. A secondo membro omogeneo
    3. Del tipo y’=y(ax+by+c)
    4. Del tipo y’=y(ax+by+c)/(a’x+b’y+c’)
    5. Equazioni differenziali esatte
    6. Mancanti delle variabili x e y
    7. Cenni sullo studio di un’assegnata equazione differenziale non lineare
    8. Di Riccati
    9. Cambi di variabile: simmetrie di Lie
  22. Analisi complessa
    1. Fondamenti
    2. Funzioni olomorfe
    3. Integrale di Cauchy e applicazioni
    4. Teorema della curva di Jordan e teorema fondamentale dell’Algebra
    5. Teorema di inversione di Lagrange
    6. Teorema dei Residui
    7. Funzioni meromorfe
    8. Prodotti infiniti e prodotti di Weierstrass
    9. Continuazione analitica e topologia
    10. Teoremi di rigidità di funzioni olomorfe
    11. Trasformata di Mellin
  23. Equazioni alle derivate parziali
    1. Equazioni del primo ordine
    2. Equazioni del secondo ordine lineari
    3. Equazioni non-lineari
    4. Sistemi di PDE
  24. Funzioni speciali
    1. Funzione Gamma di Eulero
    2. Funzioni Beta,Digamma,Trigamma
    3. Integrali ellittici
    4. Funzioni di Bessel
    5. Funzione zeta di Riemann e funzioni L di Dirichlet
    6. Funzione polilogaritmo
    7. Funzioni ipergeometriche
  25. Analisi funzionale
    1. Misura e integrale di Lebesgue
    2. Spazi Lp,teoremi di completezza e compattezza
    3. Spazi di Hilbert,serie e trasformata di Fourier
    4. Teoria e pratica dei polinomi ortogonali
    5. Spazi di Sobolev
  26. Complementi
    1. Curiosità e approfondimenti
    2. Compiti di analisi
    3. Esercizi avanzati analisi
  27. Funzioni Convesse

 
 

Tutti gli esercizi di geometria

In questa sezione vengono raccolti molti altri esercizi che coprono tutti gli argomenti di geometria proposti all’interno del sito con lo scopo di offrire al lettore la possibilità di approfondire e rinforzare le proprie competenze inerenti a tali argomenti.

Strutture algebriche.





 
 

Risorse didattiche aggiuntive per approfondire la matematica

Leggi...

  • Math Stack Exchange – Parte della rete Stack Exchange, questo sito è un forum di domande e risposte specificamente dedicato alla matematica. È una delle piattaforme più popolari per discutere e risolvere problemi matematici di vario livello, dall’elementare all’avanzato.
  • Art of Problem Solving (AoPS) – Questo sito è molto noto tra gli studenti di matematica di livello avanzato e i partecipanti a competizioni matematiche. Offre forum, corsi online, e risorse educative su una vasta gamma di argomenti.
  • MathOverflow – Questo sito è destinato a matematici professionisti e ricercatori. È una piattaforma per domande di ricerca avanzata in matematica. È strettamente legato a Math Stack Exchange ma è orientato a un pubblico con una formazione più avanzata.
  • PlanetMath – Una comunità collaborativa di matematici che crea e cura articoli enciclopedici e altre risorse di matematica. È simile a Wikipedia, ma focalizzata esclusivamente sulla matematica.
  • Wolfram MathWorld – Una delle risorse online più complete per la matematica. Contiene migliaia di articoli su argomenti di matematica, creati e curati da esperti. Sebbene non sia un forum, è una risorsa eccellente per la teoria matematica.
  • The Math Forum – Un sito storico che offre un’ampia gamma di risorse, inclusi forum di discussione, articoli e risorse educative. Sebbene alcune parti del sito siano state integrate con altri servizi, come NCTM, rimane una risorsa preziosa per la comunità educativa.
  • Stack Overflow (sezione matematica) – Sebbene Stack Overflow sia principalmente noto per la programmazione, ci sono anche discussioni rilevanti di matematica applicata, specialmente nel contesto della scienza dei dati, statistica, e algoritmi.
  • Reddit (r/Math) – Un subreddit popolare dove si possono trovare discussioni su una vasta gamma di argomenti matematici. È meno formale rispetto ai siti di domande e risposte come Math Stack Exchange, ma ha una comunità attiva e molte discussioni interessanti.
  • Brilliant.org – Offre corsi interattivi e problemi di matematica e scienza. È particolarmente utile per chi vuole allenare le proprie capacità di problem solving in matematica.
  • Khan Academy – Una risorsa educativa globale con lezioni video, esercizi interattivi e articoli su una vasta gamma di argomenti di matematica, dalla scuola elementare all’università.






Document









Document